Breast Reconstruction Flashcards

1
Q

A 54-year-old woman undergoes bilateral immediate tissue expander–based breast reconstruction. BMI is 36 kg/m2. On postoperative day 10, examination shows bilateral breast erythema; empiric antibiotics are started. In addition to Staphylococcus species, which of the following bacteria should be treated as the next most likely pathogen?

A) Bacteroides fragilis
B) Enterococcus faecalis
C) Mycobacterium marinum
D) Pasteurella multocida
E) Pseudomonas aeruginosa

A

The correct response is Option E.

After Staphylococcus aureus and S. epidermidis, Pseudomonas aeruginosa is among the the next most common sources of breast infections. It is common in hospitalized or immunocompromised patients, as well as patients with foreign body devices such as catheters or implants. It is a gram negative rod, and common antibiotic treatments include advanced β-lactams (piperacillin, ceftazidime), carbapenems, quinolones, and aminoglycosides. Dual coverage is often recommended in severe infections. In the case of breast implant infections, if there is not rapid improvement on antibiotic therapy, or if significant systemic symptoms develop (vital sign instability, high white blood cell count, fever, renal impairment), then surgical washout and device removal is mandatory. In patients without systemic symptoms, wash out and new implant placement can be an option in carefully selected and counseled patients. Other breast pathogens include Escherichia coli, Propionibacterium, and Corynebacterium.

More than 300,000 breast implant procedures are performed each year in the United States. In reconstructive cases, the infection rate averages 6% and the explantation rate 3% (range, 1.5 to 8%). Preventative measures include proper patient selection, preoperative MRSA management when carriers are suspected, routine presurgery chlorhexidine washes, proper antibiotic timing presurgery and continuation of antibiotics in implant reconstruction cases for at least 24 hours (though the optimal treatment duration has not yet been determined).

None of the other bacteria listed are common in breast infections, though all are common pathogens. Bacteroides are anaerobic gram-negative rods that are common in gut flora and feces.

Enterococcus faecalis is a frequent cause of nosocomial infection, with a high prevalence of multi-drug resistance. It is a gram-positive coccus, and is not commonly seen in breast surgery patients as it primarily colonizes the digestive tract.

Mycobaterium marinum is a rare pathogenic cause of hand infections from injuries that occur in aquatic environments.

Pasteurella multocida is a frequent cause of animal bite infections, particularly from cats and dogs.

2018

How well did you know this?
1
Not at all
2
3
4
5
Perfectly
2
Q

A 62-year-old woman is evaluated because of a new 2 × 2-cm open area near her left axillary fold. Medical history is significant for left breast cancer previously treated with bilateral mastectomies, left axillary node dissection, and adjuvant chemoradiation 10 years ago. A photograph is shown. She has been compliant with postoperative oncologic surveillance and has had no recent trauma. Which of the following underlying conditions is most likely responsible for her current presentation?

A) Empyema with spontaneous drainage
B) Lymphedema drainage tract
C) Osteoradionecrosis of the underlying rib(s)
D) Recurrent breast cancer
E) Skin ulceration from intertriginous shearing forces

A

The correct response is Option C.

The effects of ionizing radiation are permanent and may present either acutely or in delayed fashion, even years after the original radiation insult. The mechanism of injury from this radiation is through free radical production which, in turn, directly damages the DNA. In the acute period, the effects of radiation may manifest themselves as erythema and edema of the skin, vasodilation with endothelial edema, and lymphatic obliteration. This eventually leads to capillary thrombosis and subsequent inadequate tissue oxygenation. Over time, nonhealing ulcers can spontaneously develop, sometimes years later.

Although recurrent cancer is always a concern in patients with a personal history of cancer, proper, regular, and thorough surveillance can often detect recurrences early, especially in compliant patients. Most recurrences occur within the first 5 years.

Abscesses usually present initially with pain, erythema, and localized fluctuance, and often with associated fever and/or malaise. Spontaneous necessitation to the skin would also result in purulent drainage.

Lymphedema can be a chronic condition after mastectomy and axillary node dissection, and is usually manifested as generalized edema of the ipsilateral upper extremity. Sinus tract formation is rare.

Intertriginous shearing would most often present as superficial epidermal loss with possible superinfection with yeast due to moisture.

2018

How well did you know this?
1
Not at all
2
3
4
5
Perfectly
3
Q

A 48-year-old woman is evaluated for bilateral microsurgical breast reconstruction. Compared with the deep inferior epigastric perforator (DIEP) flap, the superficial inferior epigastric artery (SIEA) flap places the patient at a greater risk for which of the following complications?

A) Abdominal bulge
B) Donor site dehiscence
C) Fat necrosis
D) Flap failure
E) Umbilical necrosis

A

The correct response is Option D.

Several comparative studies have reported a higher incidence of anastomotic thrombosis and failure with the superficial inferior epigastric artery (SIEA) flap than with flaps based on the deep inferior epigastric artery (DIEA). These failure rates range from 7.35 to 14%. Most of these failures were arterial in nature. Since SIEA flaps do not require an incision into the anterior rectus sheath or rectus muscle, bulges do not occur. Reported fat necrosis rates are similar between SIEA and DIEA flaps. There is no evidence for a difference in donor site dehiscence or umbilical necrosis rates.

2018

How well did you know this?
1
Not at all
2
3
4
5
Perfectly
4
Q

A 48-year-old woman undergoes immediate unilateral breast reconstruction with a free deep inferior epigastric artery perforator (DIEP) flap. At the conclusion of the procedure, the flap skin paddle is noted to have venous congestion. Upon reexploration, the venous anastomosis appears patent with venous outflow detected by handheld pencil Doppler evaluation, but the flap continues to have venous congestion with brisk capillary refill. Which of the following is the most appropriate next step in management?

A) Apply leeches postoperatively
B) Loosely re-inset the flap and monitor closely
C) Perform a second venous anastomosis using the superficial inferior epigastric vein
D) Perform a second venous anastomosis using the vena comitans
E) Revise the venous anastomosis using a hand-sewn technique

A

The correct response is Option C.

Preservation of the superficial inferior epigastric veins (SIEV) during flap harvest is a useful preventive measure in microsurgical free tissue transfer operations. These veins can serve as important lifeboats to augment venous outflow in the setting of venous congestion. Typically, if a free flap demonstrates venous congestion, the inset should be taken down and the pedicle, recipient vessels, and anastomoses should be interrogated. Simple issues, such as mechanical compression or twisting of the vein, should be ruled out. Next, the SIEV should be inspected. If engorged, the flap is likely reliant on superficial outflow, and this vein should be connected to a recipient vessel to augment the venous outflow of the flap. Options for recipient veins include an anterograde branch on the pedicle vena comitans, or in a retrograde fashion to the vena comitans that was not used in the initial set of anastomoses.

In this case scenario, the flap continued to demonstrate venous congestion intraoperatively. This makes it unlikely that tension or pressure from the inset of the flap was causing the venous outflow obstruction. Furthermore, leech therapy is not indicated for a free flap with global venous congestion.

The venous coupling device is safe and effective for the anastomosis of veins in DIEP flap surgery. It has not been associated with patency rates that are different from hand-sewn anastomoses. The coupling device, however, has been shown to reduce the microsurgery time.

The use of one or two veins in microsurgical free tissue transfer is a topic that has been debated for several years. While some studies indicate that the use of two venous connections may reduce the velocity of blood flow across the anastomosis, there is not sufficient data to support differences in flap outcomes or thrombotic events. Therefore, the routine use of a second vein is largely up to surgeon preference.

2018

How well did you know this?
1
Not at all
2
3
4
5
Perfectly
5
Q

A 43-year-old woman undergoes the second stage of tissue expander–based breast reconstruction. Exchange of the tissue expander for a smooth round silicone implant is planned along with a superior capsulotomy and fat grafting to the upper pole for contour improvement. Which of the following is an increased risk associated with fat grafting to the breast in this patient?

A) Anaplastic large cell lymphoma
B) Benign lesions
C) Hypopigmentation
D) Infection
E) Recurrent breast cancer

A

The correct response is Option B.

Autologous fat grafting is a widely accepted technique in breast reconstruction. A large systematic review recently confirmed the oncologic safety of this technique but did report a significant incidence of benign lesions including cysts and calcifications. Fat grafting is not associated with an increased risk of recurrent breast cancer, infection or hypopigmentation.

2018

How well did you know this?
1
Not at all
2
3
4
5
Perfectly
6
Q

A 54-year-old woman with breast cancer undergoes a skin-sparing mastectomy with tissue expander reconstruction. Adjuvant chemotherapy and subsequent radiation therapy have been recommended. Chemotherapy proceeds during tissue expansion. Radiation may be performed either before or after the implant exchange procedure. When compared with radiating the permanent implant, radiating the tissue expander is most likely to increase the risk of which of the following?

A) Cancer recurrence
B) Capsular contracture
C) Device rupture
D) Explantation
E) Radiation dermatitis

A

The correct response is Option D.

Cordeiro et al. updated the largest series of women undergoing two-stage implant breast reconstruction who require postmastectomy radiation. The authors found that radiating the tissue expander, as opposed to the permanent implant, increased the rate of reconstructive failure by 46%. However, aesthetic results were better and capsular contracture was less frequent. There were no differences in patient-reported outcomes. This study confirmed the earlier findings of Nava et al. regarding reconstruction failure (explantation). There are no known differences in cancer recurrence between the two approaches discussed.

2018

How well did you know this?
1
Not at all
2
3
4
5
Perfectly
7
Q

A 52-year-old woman comes to the office to discuss revision of breast reconstruction following mastectomy for breast cancer. She is undergoing adjuvant treatment with an agent that interferes with her body’s natural mechanisms that promote native breast growth, but she cannot remember its name. She is most likely being treated with which of the following agents?

A) Alkylating agent (cyclophosphamide)
B) Anthracycline (doxorubicin)
C) Aromatase inhibitor (anastrozole)
D) Platinum agent (cisplatin)
E) Taxane (paclitaxel)

A

The correct response is Option C.

Aromatase inhibitors such as anastrozole impair conversion of androgens to estrogens. Estrogens promote normal breast tissue growth as well as growth of many breast cancers. The other options (taxanes, anthracyclines, alkylating agents, and platinum agents) are all chemotherapeutic agents that do not particularly target hormones involved in normal breast growth mechanisms.

2018

How well did you know this?
1
Not at all
2
3
4
5
Perfectly
8
Q

Improvement in which of the following is an advantage of nipple-areola complex reconstruction?

A) Breast feeding ability
B) Breast mound shape
C) Reactivity to touch
D) Satisfaction with reconstruction
E) Sexual sensation

A

The correct response is Option D.

Satisfaction with reconstruction, quality of life, and feeling of completeness with reconstruction have all been shown to be linked to nipple and/or areola reconstruction. The reconstruction does not restore nipple function; therefore, breast feeding, sexual sensation, and reactivity to touch are not accomplished with nipple and/or areola reconstruction. Often the nipple reconstruction, if using local tissue techniques, may actually flatten the anterior aspect of the breast and is not thought to positively impact the breast mound shape.

2018

How well did you know this?
1
Not at all
2
3
4
5
Perfectly
9
Q

A 35-year-old woman with a Stage T2 infiltrating ductal carcinoma is scheduled to undergo a skin-sparing, right total mastectomy and a nipple-sparing, left prophylactic mastectomy. The possibility of adjuvant radiation therapy to the right breast depends on the final surgical pathology. The patient has a history of smoking. BMI is 28 kg/m2. She wears a brassiere with a D cup and would like the postoperative result to be of a similar size. Which of the following immediate bilateral reconstructive techniques is most appropriate for this patient?

A) Abdominal-based free flaps
B) Gluteal-based free flaps
C) Latissimus dorsi myocutaneous flaps and silicone implants
D) Silicone implants and acellular dermal matrix
E) Tissue expanders and acellular dermal matrix

A

The correct response is Option E.

For this patient in whom postoperative radiation therapy is possible, the best first-stage, immediate reconstructive approach is placement of tissue expanders with acellular dermal matrix. The outcome of immediate autologous flap reconstruction may be compromised if subjected to adjuvant radiation therapy and is best delayed until after such treatment has been rendered. Although successful, cost-effective outcomes are possible with a single-stage, direct-to-implant approach, this patient has risk factors for early revision and implant failure due to her large breasts and history of smoking.

2018

How well did you know this?
1
Not at all
2
3
4
5
Perfectly
10
Q

After a nipple-sparing mastectomy, which of the following branches of an intercostal nerve predominantly provides remaining sensation to the nipple-areola complex?

A) Anterior branch of the fifth
B) Anterior branch of the fourth
C) Lateral branch of the fifth
D) Lateral branch of the fourth
E) Lateral branch of the third

A

The correct response is Option B.

The cutaneous innervation of the female breast is derived medially from the anterior cutaneous branches of the first to sixth intercostal nerves and laterally from the lateral cutaneous branches of the second to seventh intercostal nerves. The nipple-areola complex is physiologically innervated by the lateral and anterior branches of the third to fifth intercostal nerves. The fourth intercostal nerve has further shown to be most consistent in various anatomical studies.

However, the anterior branches take a superficial course within the subcutaneous tissues of the medial breast while the lateral branches take a deep course within the pectoral fascia and reach the nipple via the breast parenchyma. Therefore, the lateral branches are most likely resected during mastectomy and contribute little to the postoperative innervation of the nipple-areola complex.

2018

How well did you know this?
1
Not at all
2
3
4
5
Perfectly
11
Q

A 65-year-old woman with a history of left mastectomy for breast cancer 10 years ago undergoes biopsy of a suspicious lesion of the right breast found on a recent mammogram. Examination of the biopsy specimen confirms a right breast carcinoma. This lesion most likely originated from which of the following structures?

A) Adipose tissue
B) Areolar skin
C) Lactiferous duct
D) Lymph node
E) Pectoralis major muscle

A

The correct response is Option C.

Women who were previously treated for breast cancer are at increased risk for development of a metachronous lesion of the contralateral breast. Cancers of the breast are typically adenocarcinomas, arising from the glandular tissue such as the ducts or lobules. Paget disease of the breast would involve the areolar skin but is fairly uncommon. Sarcomas arising from the connective tissue (such as adipose or muscle) are also rare. Breast adenocarcinomas do not originate from lymphatic tissue.

2018

How well did you know this?
1
Not at all
2
3
4
5
Perfectly
12
Q

A 46-year-old woman with cancer of the right breast comes to the office to discuss a right mastectomy with immediate implant reconstruction and a symmetry procedure for the contralateral breast. Physical examination shows bilateral Grade 3 ptosis with volume symmetry. The patient currently wears a size 34C brassiere and desires to remain the same size. Which of the following is the most appropriate procedure for the contralateral breast?

A) Augmentation mammaplasty
B) Fat injections
C) Mastopexy
D) Reduction mammaplasty

A

The correct response is Option C.

Contralateral breast procedures are frequently necessary to achieve symmetry following mastectomy and reconstruction. Options to achieve symmetry include mastopexy, reduction mammaplasty, or augmentation mammaplasty combined with mastopexy. Volume symmetry can be achieved through reduction mammaplasty or fat injections. Positional asymmetry of the contralateral breast and the nipple-areola complex may require correction with mastopexy. Augmentation mammaplasty with mastopexy may be indicated for the correction of volume and positional asymmetry. The most appropriate contralateral procedure in a patient with symmetric volumes of the breast and ptosis of the contralateral breast is mastopexy.

2018

How well did you know this?
1
Not at all
2
3
4
5
Perfectly
13
Q

Which of the following is most commonly associated with alloplastic breast reconstruction in the setting of adjuvant radiation therapy?

A) Decreased rates of implant rupture
B) Decreased rates of seroma
C) Increased patient satisfaction
D) Increased risk for capsular contracture

A

The correct response is Option D.

Alloplastic breast reconstruction increases the risk for capsular contracture in the setting of adjuvant radiation therapy. It can also increase the risks for seroma, wound healing complications, and infections. Radiation would not decrease the rates of implant rupture and would not increase patient satisfaction.

2018

How well did you know this?
1
Not at all
2
3
4
5
Perfectly
14
Q

A 36-year-old woman with invasive ductal carcinoma of the left breast comes to the office to discuss options for immediate breast reconstruction. Examination shows that the tumor is positive for estrogen receptor (ER) and progesterone receptor (PR) expression. Family history is negative for breast and ovarian cancer, and genetic testing shows no abnormalities. Bilateral mastectomy is planned. Which of the following outcomes is most likely in this patient following contralateral prophylactic mastectomy of the right breast?

A) Decreased number of anticipated overall operations
B) Decreased risk of recurrence of the known cancer
C) Increased cure rate for her known cancer
D) Increased possibility of future breast-feeding
E) Increased risk of surgical site complications

A

The correct response is Option E.

Contralateral prophylactic mastectomy (CPM) has become increasingly common in recent years. Women who are at higher than average risk of developing breast cancer may have a clear oncologic benefit from CPM in terms of risk reduction. This would include women who are carriers of high-risk genetic mutations such as BRCA1 and BRCA2, women with a strong family history of breast cancer, and young women with high risk of aggressive tumors. However, for women with average risk of contralateral breast cancer (2 to 6% over 10 years), studies have not supported an oncologic benefit to CPM, attitudes of the lay public and media attention notwithstanding.

Since the surgeries of each breast, involving both mastectomy and reconstruction, are largely (although not completely) independent of one another, the risk of surgical complications to one side or the other is nearly additive, resulting in an approximate doubling of those risks.

CPM has not been shown to improve the cure rate for the known cancer treated with therapeutic mastectomy.

CPM has not been shown to reduce risk of recurrence for the known cancer on the primary side.

CPM would not be expected to decrease the number of anticipated operations. Given the increased likelihood of complications, one may reasonably expect the same number of operations or greater (for treatment of complications), but not fewer.

Removal of both breasts eliminates the ability to breast-feed in the future.

2018

How well did you know this?
1
Not at all
2
3
4
5
Perfectly
15
Q

A 75-year-old woman is evaluated after undergoing bilateral autologous breast reconstruction with deep inferior epigastric artery perforator (DIEP) flaps 2 days ago. The patient had been recovering well until this morning, when she developed acute respiratory insufficiency with hypoxia, tachycardia, and tachypnea. Pulmonary embolism is suspected, and pulmonary CT angiography has been ordered. Baseline creatinine level is 1.1 mg/dL. Which of the following steps should be taken to prevent contrast-induced nephropathy in this patient?

A) Hydration with hydroxyethyl starch (colloid)
B) Hydration with 0.9% saline
C) No preventive measures are needed
D) Pretreatment with the antioxidant N-acetylcysteine
E) Pretreatment with 0.9% saline and diphenhydramine

A

The correct response is Option B.

Although the risk of contrast-induced nephropathy (CIN) is overall relatively low, geriatric patients are at higher risk for this complication secondary to their high prevalence of risk factors for renal dysfunction. In this patient population, even normal laboratory creatinine concentrations are not always indicative of normal glomerular filtration rate (GFR), and every effort should be made to prevent the development of CIN. Important prophylactic strategies to avoid this adverse effect include optimization of fluids and avoidance of dehydration, use of non-ionic contrast media, and avoiding repeated exposure to contrast at close intervals.

This geriatric patient should be well hydrated prior to undergoing exposure to this high-contrast study. Minimizing fluid and performing no intervention will increase the patient’s risk of developing CIN even with a normal creatinine concentration. Antioxidants such as N-acetylcysteine, vasodilators such as dopamine, and colloid solutions such as Hespan have not proven to be beneficial in preventing renal dysfunction. Pretreatment with 0.9% saline and diphenhydramine is used in patients who have allergies to the contrast dye to minimize the allergic reaction and plays no role in preventing CIN.

2018

How well did you know this?
1
Not at all
2
3
4
5
Perfectly
16
Q

Which of the following factors is most likely to lead to a patient undergoing breast reconstruction after mastectomy?

A) BRCA1/BRCA2 status
B) Insurance coverage through Medicaid
C) Living close to a reconstructive surgeon
D) Patient age
E) Residence in the southeastern United States

A

The correct response is Option C.

Several large population-based studies in different regions of the United States show that provision of breast reconstruction is not uniform among all mastectomy patients. The two biggest factors are geographic distance from reconstructive surgeons and insurance status. Patients who live farther away from providers of reconstruction and those uninsured or with Medicaid have a much lower incidence of reconstruction compared to the overall cohorts. Most studies also show BRCA positive and younger patients tend to have a higher incidence of bilateral reconstruction.

2018

How well did you know this?
1
Not at all
2
3
4
5
Perfectly
17
Q

Following a skin-sparing mastectomy, a 39-year-old woman undergoes deep inferior epigastric perforator (DIEP) flap breast reconstruction. To augment flap sensation, the anterior sensory branch of the fourth intercostal nerve is coapted to which of the following nerves within the DIEP flap?

A) Genitofemoral
B) Iliohypogastric
C) Ilioinguinal
D) Intercostal
E) Lateral femoral cutaneous

A

The correct response is Option D.

The third, fourth, and fifth intercostal nerves are responsible for innervation of the majority of the breast. The anterior branch of the fourth intercostal nerve provides most erogenous sensation to the nipple. Sensation to the lower abdomen arises from segmental cutaneous branches of the intercostal nerve, which travel through the rectus abdominis muscle. T10 provides sensation to the dermatome, including the periumbilical region, and is most commonly used. The iliohypogastric nerve provides sensation to the lateral gluteal region. The ilioinguinal nerve provides sensation to the upper medial thigh. The genitofemoral nerve provides sensation to the upper anterior thigh and mons pubis. The lateral femoral cutaneous nerve provides innervation to the lateral thigh and is not used for this purpose.

2017

How well did you know this?
1
Not at all
2
3
4
5
Perfectly
18
Q

Which of the following is a risk factor for hormone-sensitive breast cancer?

A) Breast-feeding
B) Early age at first pregnancy
C) Early menopause
D) Late menarche
E) Post-menopausal obesity

A

The correct response is Option E.

Post-menopausal obesity is associated with increased adipose production of estrogen, which can increase the risk for hormone-sensitive breast cancer. Other options (late menarche, early menopause, and breast feeding) decrease the number of menstrual cycles, and therefore may decrease the risk of breast cancer. Early age at first pregnancy is also associated with decreased risk for hormone-sensitive breast cancers.

2017

How well did you know this?
1
Not at all
2
3
4
5
Perfectly
19
Q

A 54-year-old woman with BRCA mutation is scheduled for a bilateral nipple-sparing mastectomy. Which of the following would lead to the highest risk for postoperative nipple necrosis?

A) Grade 2 breast ptosis
B) Periareolar mastectomy incision
C) Previous excisional breast scar
D) Tumor greater than 1.5 cm
E) Use of smooth round gel implants

A

The correct response is Option B.

Nipple-sparing mastectomy and direct to implant reconstruction is becoming increasingly popular in the setting of prophylactic mastectomies. The criteria for nipple-sparing mastectomies have been increased to tumors not involving the nipple-areola complex, with some surgeons requiring a distance of 2 cm from the nipple and others espousing just a clean surgical margin at the nipple. Nipple-sparing mastectomies do not require recreating the breast envelope and therefore direct to implant reconstruction is possible. Increased risk for incidence of nipple necrosis in the setting of direct to implant reconstruction is associated with mastectomy incision involving the areola. The use of smooth round gel implants, grade 2 ptosis, tumor size, or previous breast biopsy scars are not associated with nipple necrosis.

2017

How well did you know this?
1
Not at all
2
3
4
5
Perfectly
20
Q

An otherwise healthy 16-year-old girl comes to the office because of a painless mass in the left breast. Physical examination of the left breast discloses a circumscribed firm, rubbery, 3-cm mass without overlying skin changes, and no axillary lymphadenopathy. Results of a pregnancy test are negative. Which of the following is the most likely diagnosis?

A) Common fibroadenoma
B) Giant fibroadenoma
C) Lactating adenoma
D) Phyllodes tumor
E) Tubular adenoma

A

The correct response is Option A.

A common fibroadenoma is the most likely diagnosis of this patient. Common fibroadenoma is the most common breast tumor in adolescent females and present between the ages of 14 and 16.

Juvenile fibroadenoma is a variant of fibroadenoma and is usually seen in adolescents and young adults. It is associated with a normal stromal/epithelial balance, which distinguishes it from phyllodes tumor, and has both stromal and epithelial hyperplasia. In addition, juvenile fibroadenomas are characterized by rapid growth.

A giant fibroadenoma is a clinical diagnosis, rather than a pathologic diagnosis. It is characterized by its size, usually greater than 5 cm.

Complex fibroadenoma is characterized by fibrocystic changes on glandular tissue with underlying features of common fibroadenoma on pathologic analysis.

A tubular adenoma has glandular proliferation on pathologic analysis, and while it is a subtype of fibroadenoma, it is not as common as common fibroadenoma. A lactating adenoma, similarly, is defined by the presence of secretory hyperplasia of lobules on pathologic analysis. Lactating adenomas are so defined because of the histologic presence of secretory hyperplasia, and they commonly occur in pregnant or lactating women. Many lactating adenomas will spontaneously regress.

A phyllodes tumor is typically a large, rapidly growing lesion and can be either benign or malignant. It is rare in adolescents, but when found, is usually aggressive. Phyllodes tumor is related to fibroadenoma and is distinct from other forms of breast cancer. Treatment is wide local excision.

2017

How well did you know this?
1
Not at all
2
3
4
5
Perfectly
21
Q

A 30-year-old woman comes to the office because of pain in the left breast. Two weeks ago, she underwent core needle biopsy of a breast mass that was diagnosed as benign. Family history does not include breast cancer. She does not smoke cigarettes. On examination, the left breast is erythematous and tender to palpation, and the skin of the breast is retracted laterally. There is a palpable, rope-like mass that courses longitudinally along the breast. Which of the following is the most appropriate diagnosis and treatment of this lesion?

A) Breast abscess, perform incision and drainage of the mass
B) Fibrocystic changes, perform biopsy
C) Mastodynia, treat with 10-day course of broad-spectrum antibiotics
D) Nipple papilloma, perform diagnostic mammography
E) Superficial thrombophlebitis, manage with analgesics

A

The correct response is Option E.

Mondor disease, or superficial thrombophlebitis of the breast, involves the superficial veins of the breast and anterior chest wall. It may occur following surgery, core biopsy, irradiation, or trauma. Clinical manifestations include pain, redness and swelling, and the presence of a thickened tender cord. This condition usually resolves in 4 to 6 weeks with symptomatic treatment using pain relief.

Nipple papillomas may be identified as a mass on breast imaging or may be found incidentally. They frequently present with bloody nipple discharge. While not concerning in and of themselves, these lesions may harbor areas of atypia or ductal carcinoma in situ, and are treated with core needle biopsy.

While a breast abscess is possible after a diagnostic procedure such as a biopsy, it would present as localized swelling, tenderness, and induration. The skin retraction and rope-like mass would not be present.

Fibrocystic changes in the breast present as a solitary mass and may cause patients to seek medical attention because of associated pain. They may fluctuate in size and tenderness during a patient’s menstrual cycle. Because no breast mass can be definitively declared benign on physical examination alone, imaging and/or biopsy may be considered.

Breast pain in the absence of a finding on physical examination may have a number of causes including menstrual changes, breast hypertrophy, diet, hormone replacement therapy, ductal ectasia, mastitis, malignancy, and hidradenitis. While the patient does have breast pain, the other physical findings rule out mastodynia alone.

2017

How well did you know this?
1
Not at all
2
3
4
5
Perfectly
22
Q

Oral contraceptive use increases the incidence of which of the following histologic risk factors for breast cancer?

A) Apocrine metaplasia
B) Atypical hyperplasia
C) Intraductal hyperplasia
D) Intraductal papilloma
E) Sclerosing adenosis

A

The correct response is Option B.

Patients in whom an otherwise benign breast biopsy shows atypical hyperplasia have a 4.5- to 5-fold increased risk for developing breast cancer. Proliferative breast disease without atypia increases the risk 1.5- to 2-fold. Oral contraceptives have shown to decrease the occurrence of all proliferative forms of benign breast disease without atypia, such as intraductal hyperplasia, intraductal papilloma, and sclerosing adenosis. Apocrine metaplasia is a non-proliferative histologic change and carries no increased risk for breast cancer. Of the choices, only atypical hyperplasia is increased with oral contraceptive use.

2017

How well did you know this?
1
Not at all
2
3
4
5
Perfectly
23
Q

A 42-year-old nurse is scheduled to undergo elective non–implant-based surgery of the left breast. Medical history includes no abnormalities, and she has no allergies. She smokes 1 pack of cigarettes daily. To decrease the incidence of surgical site infection, which of the following is the most effective perioperative strategy?

A) Administer cefazolin intravenously within 5 minutes of skin incision
B) Administer an insulin drip to keep blood glucose concentration less than 100 mg/dL
C) Have the patient practice complete smoking cessation for 7 days prior to surgery
D) Prescribe nasal mupirocin and chlorhexidine baths for 5 days prior to surgery
E) Use povidone-iodine skin preparation rather than chlorhexidine/isopropyl alcohol

A

The correct response is Option D.

A 2010 randomized controlled trial of over 6700 patients published in the New England Journal of Medicine documented a nearly 60% decrease in Staphylococcus aureus infections in patients if mupirocin was applied twice a day to the nares as well as a full-body wash with chlorhexidine for 5 days prior to surgery. The mean hospital stay was already reduced almost 2 days. A meta-analysis also demonstrated the same findings, with a nearly 45% decrease in surgical site infections (SSIs).

Data would suggest that isopropyl alcohol–containing skin preparations for surgery decrease SSI rates more effectively than povidone-iodine alone.

Perioperative antibiotics should be administered with enough advance time to achieve proper and adequate rates of skin penetration. With cefazolin, this is 30 to 59 minutes before skin incision, with an odds ratio of 1.0 (vs. 2.0 if given within 30 minutes, and 1.7 if given after 60 minutes). Five minutes prior to skin incision is not sufficient for the SSI-reducing effect to be achieved.

Blood glucose control is critical to decreasing SSI rates, with optimal rates usually being quoted as less than 180 mg/dL. However, this patient is not diabetic; she is otherwise healthy, so an insulin drip would not be appropriate. In addition, hypoglycemia can also have detrimental physiologic effects and also should be avoided.

Smoking cessation decreases SSI rates if the patient does not smoke for 4 weeks before or after surgery. A 2012 systematic review and meta-analysis of nearly 500,000 patients demonstrated this. However, 7 days of smoking cessation is insufficient time in advance of surgery to obtain these statistically significant benefits.

2017

How well did you know this?
1
Not at all
2
3
4
5
Perfectly
24
Q

A 45-year-old woman comes to the office for consultation regarding severe breast asymmetry after undergoing lumpectomy and radiation therapy for ductal carcinoma of the right breast 5 years ago. Physical examination shows the radiated right breast is tight and retracted, and the left breast is ptotic. Left-sided mastopexy for symmetry and autologous fat grafting to the radiated breast are recommended. The patient asks if the procedure will be covered by insurance. Which of the following is the most appropriate response?

A) Fat grafting the right breast will be covered by insurance but the mastopexy will be considered cosmetic and will not be covered
B) Insurance companies rarely cover the cost of immediate breast reconstruction
C) Insurance may not cover the procedure since insurers are not mandated to pay for reconstruction of lumpectomy defects
D) Procedures for both sides will be covered by insurance
E) The left-sided mastopexy will likely be covered but the fat grafting will not be covered

A

The correct response is Option C.

The Women’s Health and Cancer Rights Act, signed into law in 1998, requires insurance plans to cover the cost of breast reconstruction after mastectomy. The law includes all stages of reconstruction as well as contralateral procedures to provide symmetry. An often misunderstood aspect of the Women’s Health and Cancer Rights Act is that it does not apply to individuals undergoing breast conservation therapy (lumpectomy with radiation). As rates of breast conservation therapy have continued to increase (60% of women with stage I cancers), so have significant lumpectomy defects associated with the untoward effects of radiation. In the clinical scenario presented, the patient should be informed that the corrective operation (fat grafting and mastopexy) might not be covered by her insurance company.

2017

How well did you know this?
1
Not at all
2
3
4
5
Perfectly
25
Q

A 35-year-old woman comes to the office to discuss a recent diagnosis of breast cancer. Recent mammography showed diffuse microcalcifications throughout the breast, and needle biopsy showed infiltrating ductal carcinoma. On physical examination, some retraction of the skin in the lower outer quadrant of the breast is noted. She wears a size 36C brassiere. The patient reports that she is currently considering whether to have lumpectomy and radiation therapy or mastectomy. Which of the following features of this clinical scenario is a CONTRAINDICATION to breast conservation therapy?

A) Breast size
B) Mammographic findings
C) Patient age
D) Skin retraction on physical examination of the breast
E) Tumor pathology

A

The correct response is Option B.

Breast conservation therapy (BCT) refers to breast conserving therapy followed by moderate-dose radiation to eradicate microscopic residual disease. The goal is to provide the equivalent survival of mastectomy while maintaining a cosmetically acceptable appearance with a low rate of recurrence.

When considering breast conservation therapy or mastectomy, the needs and desires of each patient should be addressed. Age alone is not a contraindication to BCT, but overall health and comorbidities should be considered. Histologic subtype and pathology are not contraindications to BCT as long as the tumor is not diffuse and can be safely excised with negative margins. Similarly, breast size needs to be considered along with tumor size and location, but “small” or “large” breasts are not indications or contraindications. While retraction of the skin, nipple, or breast parenchyma is not an absolute contraindication to BCT, as long as negative margins can be safely removed, the cosmetic impact of their involvement should be considered.

There are few absolute contraindications to BCT, but they include:

Multicentric disease with two or more tumors in separate quadrants of the breast such that they cannot be encompassed in a single excision

Diffuse malignant microcalcifications on mammography

A history of prior radiation in the same breast or chest wall

Pregnancy

Persistently positive margins despite re-excision

2017

How well did you know this?
1
Not at all
2
3
4
5
Perfectly
26
Q

A 52-year-old woman undergoes immediate unilateral breast reconstruction with a free deep inferior epigastric flap. Near completion of the procedure, the flap skin paddle is noted to have venous congestion. On exploration of the pedicle, the anastomosis between the vena comitans and the internal mammary vessel appears patent. Which of the following preventive measures would most likely have averted this issue?

A) Administration of heparin immediately before release of vessel clamps and flap revascularization
B) Anastomosis of the veins using sutures instead of a venous coupling device
C) Preservation and anastomosis of the superficial inferior epigastric vein
D) Routine anastomosis of two venae comitantes per flap
E) Use of near-infrared fluorescence imaging to assess flap blood flow

A

The correct response is Option C.

Preservation of the superficial inferior epigastric veins (SIEV) during flap harvest is a useful preventive measure in microsurgical free tissue transfer operations. These veins can serve as important lifeboats to augment venous outflow in the setting of venous congestion. Typically, if a free flap demonstrates venous congestion, the inset should be taken down and the pedicle, recipient vessels, and anastomoses interrogated. Simple issues, such as mechanical compression or twisting of the vein should be ruled out. Next, the SIEV should be inspected. If it is engorged, it is likely that the flap is reliant on superficial outflow, and this vein should be connected to a recipient vessel to augment the venous outflow of the flap. Options for recipient veins include an anterograde branch on the pedicle vena comitans, or in a retrograde fashion to the vena comitans that was not used in the initial set of anastomoses.

Near-infrared fluorescence imaging technology can assist with flap design and may be useful for assessing the arterial inflow of the flap, but it has not been shown to correlate with flap loss or venous complications.

The use of one or two veins in microsurgical free tissue transfer is a topic that has been debated for several years. While some studies indicate that the use of two venous connections may decrease the velocity of blood flow across the anatomosis, there are no data to support differences in flap outcomes or thrombotic events. Therefore, the routine use of a second vein is largely up to surgeon preference.

Heparin may be a useful adjunct when thrombosis of the arterial or venous anastomosis is excised and a revision of the anastomosis is performed. Without evidence of thrombosis, it is unlikely to have any added benefit.

For venous anastomoses, the use of venous coupling devices has not been associated with patency issues or increased thrombosis rates. Therefore, a hand-sewn anastomosis is not likely to prevent the issue presented in this question.

2016

How well did you know this?
1
Not at all
2
3
4
5
Perfectly
27
Q

A 36-year-old woman with genetic susceptibility to breast cancer is scheduled to undergo bilateral prophylactic mastectomy. She has elected to proceed with immediate single-stage reconstruction using permanent silicone implants and acellular dermal matrix. Which of the following is the most sensitive method to detect implant rupture in this patient?

A) CT scan
B) Mammography
C) MRI
D) Physical examination
E) Ultrasonography

A

The correct response is Option C.

Implant rupture is one of the most common reasons for implant removal. While implant rupture can be associated with symptoms such as capsular contracture, it is often completely asymptomatic or “silent.” For this reason, screening imaging to detect such ruptures has been recommended.

According to the 2011 Update on the Safety of Silicone Gel-Filled Breast Implants published by the U.S. Food and Drug Administration, it is currently recommended that women with silicone implants get their first MRI 3 years after they receive the implants and every 2 years thereafter to detect silent ruptures.

Since the rate of rupture increases the longer an implant is in place, not screening is unacceptable.

Mammograms can detect extracapsular silicone when an implant ruptures, but they do not detect intracapsular ruptures.

The accuracy of ultrasound largely depends on the skill of the ultrasound technologist, the type of equipment used, and the experience of the interpreting physician. Furthermore, ultrasound is limited in its ability to detect ruptures in the back wall of the implant and in the breast tissue behind it.

CT scans can detect intracapsular silicone gel-filled breast implant rupture, but they are limited in their ability to detect extracapsular ruptures.

2016

How well did you know this?
1
Not at all
2
3
4
5
Perfectly
28
Q

A 53-year-old woman comes to discuss breast reconstruction after undergoing left mastectomy for ductal carcinoma in situ. She does not require chemotherapy or radiation therapy and does not want surgery on the unaffected breast. She is obese but otherwise healthy with a large, ptotic right breast. Which of the following breast reconstruction techniques is most likely to result in the greatest long-term patient satisfaction?

A) Autologous breast reconstruction with microvascular free tissue transfer from the abdomen
B) Extended latissimus dorsi musculocutaneous flap reconstruction without an implant
C) Immediate reconstruction with cohesive, anatomically shaped silicone gel implant
D) Immediate tissue expander followed by implant-based reconstruction
E) Latissimus dorsi musculocutaneous flap reconstruction with immediate placement of a permanent implant

A

The correct response is Option A.

Multiple studies have supported improved patient satisfaction with autologous breast reconstruction in the setting of a unilateral reconstruction. The clinical scenario involves an obese patient with a large, ptotic breast who does not desire a surgical procedure for symmetry and is the ideal candidate for an autologous reconstruction. Microvascular free tissue transfer of abdominal tissue for breast reconstruction demonstrates improved reliability and decreased fat necrosis compared with pedicled flap reconstruction. Implant-based reconstruction is less likely to provide adequate symmetry in this patient. An extended latissimus dorsi musculocutaneous flap–based reconstruction alone is unlikely to provide enough volume for symmetry and a traditional latissimus flap with implant is unlikely to provide adequate ptosis for symmetry.

2016

How well did you know this?
1
Not at all
2
3
4
5
Perfectly
29
Q

A 43-year-old woman would like to discuss plans for breast reconstruction after her upcoming unilateral mastectomy. Postoperative radiation therapy is planned. Which of the following is the most likely benefit of tissue expander–based breast reconstruction compared with immediate autologous breast reconstruction using this patient’s abdominal tissue?

A) Better symmetry
B) Improved postoperative sensation
C) A larger, more ptotic breast reconstruction
D) Lower risk of complications from radiation
E) Preservation of the patient’s options for final reconstruction

A

The correct response is Option E.

Immediate breast reconstruction with tissue expanders followed by reconstruction of choice preserves the patient’s skin envelope and keeps open options for definitive final reconstruction of choice whether with an implant or autologous tissue.

Tissue expander-based reconstruction is associated with a higher complication rate in the setting of radiation therapy but preserves abdominal and back tissue as options for autologous reconstruction. Implant-based reconstruction does not provide the advantages of improved symmetry, sensation, or breast ptosis.

2016

How well did you know this?
1
Not at all
2
3
4
5
Perfectly
30
Q

A 62-year-old woman undergoes breast reconstruction using autologous tissue from the abdomen. Intraoperatively, use of a perforator flap is found to be impossible because of multiple small nondominant perforators. Conversion to a delayed pedicled transverse rectus abdominis muscle flap is planned. Ligating which of the following vessels in this stage will best facilitate future viability of the tissue transferred in the next stage?

A) Deep inferior epigastric
B) Hypogastric
C) Internal mammary
D) Superficial inferior epigastric
E) Superior epigastric

A

The correct response is Option A.

Pedicled transverse rectus abdominis muscle (TRAM) flaps are based on the superior epigastric system, which is often less robust than the deep inferior epigastric system. Therefore, surgical delay by ligation of the deep inferior epigastric system may facilitate overall viability of the transferred tissue. Ligation of the superior epigastric system would make a pedicled TRAM flap unlikely to survive. The internal mammary ligation may also interrupt blood supply to the superior epigastric system, and even if the tissue is fed through collaterals, it would not strengthen the flap. Division of the superficial inferior epigastric system might also help, but it is not as critical as ligation of the deep inferior epigastric system. The hypogastric system does not have a direct impact on the pedicled TRAM tissues.

2016

How well did you know this?
1
Not at all
2
3
4
5
Perfectly
31
Q

A 35-year-old BRCA-positive woman undergoes bilateral prophylactic nipple-sparing mastectomy via lateral radial incisions with periareolar extensions. She undergoes direct-to-implant reconstruction with an acellular dermal matrix inferior lateral sling. Mastectomy specimens weigh 435 g each, and placement of 450 mL smooth, round, high-profile implants is performed. Postoperatively, 25% partial nipple areolar complex necrosis in both breasts is noted. Which of the following most likely contributed to the necrosis?

A) Bilateral surgery
B) Mastectomy incision
C) Use of acellular dermal matrix
D) Volume of the implants
E) Weight of the mastectomy specimen

A

The correct response is Option B.

Nipple-sparing mastectomies are becoming more prevalent with the rise of prophylactic mastectomies and increased comfort with performing it in presence of in situ and invasive carcinoma. While universal standards for patient selection do not exist, there is consensus that general recommendations include no inflammatory breast cancers, no pathologic nipple discharge, no Paget’s disease, and for many, a 2-cm distance from the tumor to the nipple. Nipple-sparing mastectomy incisions have been described as radial lateral, lateral inframammary fold, and peri-areolar. Because the entire skin envelope is preserved, direct-to-implant reconstruction is possible with inferolateral support from acellular dermal matrices or other scaffolds. Areolar involvement in mastectomy incisions is associated with increased rates of nipple necrosis. The weight of mastectomy specimen, volume of implants, use of acellular dermal matrices or laterality of surgery have not been shown to be associated with nipple necrosis.

2016

How well did you know this?
1
Not at all
2
3
4
5
Perfectly
32
Q

An otherwise healthy 38-year-old woman who is a smoker is considering implant-based breast reconstruction following mastectomy. She has been counseled about likely use of acellular dermal matrix when the intermediate tissue expander is placed and wants to further understand why the matrix will be used in her body. The patient should be advised that use of acellular dermal matrix is associated with a decreased risk for which of the following?

A) Cancer recurrence
B) Capsular contracture
C) Seroma
D) Skin flap necrosis
E) Smoking-related complications

A

The correct response is Option B.

Acellular dermal matrix (ADM) use has been increasing over time with tissue expander or implant-based breast reconstruction following mastectomies. Many potential advantages and disadvantages have been studied, and some of the data are contradictory. However, the consensus of the literature indicates that ADMs are associated with decreased capsular contracture rates. There is literature to suggest that seroma rates are increased or remain stable, not decreased, with ADMs. ADMs have not been shown to decrease independent patient risk factors for complications such as tobacco use or to decrease cancer recurrence rates. ADMs also do not appear to improve vascularity of the tissue overlying them when initially placed.

2016

How well did you know this?
1
Not at all
2
3
4
5
Perfectly
33
Q

A 45-year-old woman undergoes mastectomy. Which of the following anatomical landmarks denotes a limit of the breast and, therefore, a limit to the extent of the mastectomy?

A) Anterior border of latissimus dorsi muscle
B) Inferior origin of pectoralis major muscle
C) Lateral pectoralis minor muscle
D) Superficial fascia of the serratus anterior muscle
E) Supraclavicular lymph node basin

A

The correct response is Option A.

The borders of the breast and, therefore, the limits of the mastectomy are the sternum, clavicle, inframammary fold, and anterior border of the latissimus dorsi muscle. The other options are not designated as borders of the breast tissue.

2016

How well did you know this?
1
Not at all
2
3
4
5
Perfectly
34
Q

A 57-year-old woman with a history of modified left radical mastectomy followed by adjuvant chemotherapy and radiation therapy is evaluated six months after her last radiation treatment for breast reconstruction. Physical examination shows slight skin redundancy of the chest wall and grade 2 ptosis of the contralateral breast. She wears a size 36D brassiere and does not want to undergo symmetry procedures on the contralateral breast. Which of the following reconstructive options is most likely to achieve breast symmetry in this patient?

A) Deep inferior epigastric perforator flap
B) Latissimus dorsi flap
C) Thoracodorsal artery perforator flap
D) Tissue expander with acellular dermal matrix and staged implant placement
E) Total submuscular tissue expander and staged implant placement

A

The correct response is Option A.

Breast reconstruction requires reconstruction of both the soft-tissue envelope and volume. In the setting of radiated chest wall tissues, it would be difficult to recreate a breast envelope of sufficient size to match the contralateral breast with tissue expansion, even with acellular dermal matrix or total submuscular placement of a tissue expander. The history of radiation would likely cause an element of capsular contracture and decrease symmetry to the contralateral breast with grade 2 ptosis. Both the latissimus dorsi flap and the thoracodorsal artery perforator flap would recruit non-radiated skin to create an adequate envelope, but would not provide adequate volume alone. The deep inferior epigastric perforator (DIEP) flap would provide both envelope and volume.

2016

How well did you know this?
1
Not at all
2
3
4
5
Perfectly
35
Q

A 53-year-old woman with a BMI of 27 kg/m2, gigantomastia, and grade III ptosis is considering unilateral mastectomy and autologous reconstruction with abdominal tissue. Which of the following is the most appropriate advice regarding the contralateral breast?

A) A contralateral prophylactic mastectomy should be performed to maximize symmetry
B) Contralateral reduction should be accompanied by placement of an implant on that side to address upper pole symmetry with the autologous reconstruction
C) Simultaneous contralateral reduction may be performed with an acceptable risk profile
D) Symmetry is unlikely to be achieved, so a contralateral matching procedure would not be recommended
E) Symmetry with autologous tissue is likely to be achieved without the contralateral matching procedure

A

The correct response is Option C.

Studies have shown that contralateral symmetry procedures performed synchronously with unilateral autologous tissue reconstruction after mastectomy (including reconstructions with a free perforator flap) are acceptable. Situations requiring reduction rather than augmentation or mastopexy seem to be the most suited to this timing. A patient with a lower BMI and gigantomastia is unlikely to achieve symmetry without a contralateral procedure. Autologous tissue reconstruction likely would not need a contralateral implant for upper pole symmetry. An attempt at improvement in symmetry using contralateral surgery would be acceptable should the patient so choose, but recommending a prophylactic mastectomy on the contralateral side solely for symmetry and not for risk reduction may be overly aggressive when other methods such as reduction mammaplasty exist with a likelihood of acceptable postoperative symmetry.

2016

How well did you know this?
1
Not at all
2
3
4
5
Perfectly
36
Q

Which of the following is the most accurate location of the elliptical skin island of a profunda artery perforator (PAP) flap?

A) Anteromedial thigh with the superior border within the groin crease
B) Inferior buttock with the inferior border within the gluteal fold
C) Lateral hip superior to the iliac crest
D) Middle buttock, from the posterior superior iliac spine to the apex of the greater trochanter
E) Posteromedial thigh with the superior border within the gluteal fold

A

The correct response is Option E.

According to the literature, the skin island of the profunda artery perforator (PAP) flap is inferior to the gluteal fold.1,2 An ellipse of the inferior buttock with the inferior border within the gluteal fold describes the skin island of the inferior gluteal artery perforator (IGAP) free flap.3 An ellipse of the anteromedial thigh with the superior border within the gluteal fold describes the transverse upper gracilis (TUG) flap. An ellipse of the middle buttock, from the posterior superior iliac spine to the apex of the greater trochanter, describes the superior gluteal artery perforator (SGAP) flap.3 An ellipse of the lateral hip superior to the iliac crest describes the Rubens or lateral hip flap.4 The only option that correctly identifies the skin island for the PAP flap is an ellipse of the posteromedial thigh with the superior border within the gluteal fold. The superior marking is within or just below the gluteal fold and the inferior marking is roughly 7 cm below the superior marking. The flap is an ellipse so the scar does not extend outside of the gluteal fold.1,2

2016

How well did you know this?
1
Not at all
2
3
4
5
Perfectly
37
Q

Nipple-sparing mastectomy is CONTRAINDICATED in which of the following women considering mastectomy and immediate breast reconstruction?

A) A 21-year-old with BRCA1 mutation and grade I breast ptosis
B) A 31-year-old with BRCA2 mutation; subglandular augmentation mammaplasty 1 year ago
C) A 45-year-old with unilateral breast cancer and need for postoperative radiation therapy
D) A 50-year-old with unilateral breast cancer in the tail of Spence
E) A 61-year-old with unilateral breast cancer; tumor-to-nipple distance of 1 cm

A

The correct response is Option E.

Nipple-sparing mastectomy is increasingly prevalent owing to perceived improvement in reconstructive outcome and patient satisfaction. Prevalence of nipple ischemia, which can vary in severity from incomplete, partial-thickness epidermolysis to total nipple loss, ranges from 2 to 60% depending on the definition used. A recent pooled analysis found a cumulative prevalence of 7%. Most cases respond to local wound care; reoperation for total nipple loss is relatively infrequent.

Patient selection remains paramount to successful nipple preservation. Typically, thin, non-smoking patients with small, non-ptotic breasts are considered ideal candidates. Severe macromastia and ptosis may not only increase risk of poor nipple vascularity, but also contribute to nipple malposition. Oncologic determinants of the safety of nipple preservation should also be considered. Noninflammatory cancers located in excess of 2 cm from the nipple can generally be safely extirpated without removal of the nipple. Usually, an intraoperative frozen section biopsy of the retroareolar tissue (“doughnut”) is performed after excision of the main mastectomy specimen to demonstrate nipple margins free of tumor. Patients undergoing prophylactic mastectomy in the setting of high genetic predisposition are also considered good candidates. Successful nipple preservation has also been described in patients with a history of reduction mammaplasty or mastopexy.

Successful nipple-sparing mastectomy with implant reconstruction has been described recently in patients who require postoperative radiation, albeit with higher risk of capsular contracture and nipple malposition. Prior augmentation in the subglandular or subpectoral positions does not contraindicate nipple-sparing mastectomy; rather, the latter group may present opportunities for direct-to-implant reconstruction. Location of a breast cancer in the tail of Spence (axillary extension of breast tissue) should not contraindicate nipple-sparing mastectomy, as it is likely to be far enough from the nipple to safely spare it.

2016

How well did you know this?
1
Not at all
2
3
4
5
Perfectly
38
Q

A 62-year-old woman with a history of Stage III breast cancer is scheduled for delayed autologous breast reconstruction from the abdominal donor site. She has no other medical problems. BMI is 30 kg/m2. Her mother had a lower extremity deep venous thrombosis in the past. Caprini risk assessment score is 9. Which of the following is the most appropriate method of postoperative VTE risk reduction?

A) Aspirin therapy
B) Early ambulation after surgery
C) Low-molecular-weight heparin therapy
D) Sequential compression device use
E) No VTE prevention is indicated

A

The correct response is Option C.

Venous thromboembolism (VTE) is a disorder with short-term mortality and long-term morbidity. Plastic and reconstructive surgery patients are known to be at high risk for VTE after surgery. Symptomatic VTE occurs with high frequency after post-bariatric body contouring (7.7%), abdominoplasty (5%), and breast or upper body contouring (2.9%). To fully identify VTE risk in surgical patients, individualized patient assessment is advocated. The Caprini risk assessment model (RAM) is a useful and effective tool to stratify surgical patients for VTE risk. For patients with high Caprini scores, a significantly greater likelihood of VTE events is observed. Approximately 11% of patients with Caprini score >8 will have a VTE within 60 days after surgery.

Based upon recommendations from the ASPS VTE Task Force, patients undergoing elective plastic and reconstructive surgical procedures who have Caprini RAM score of 7 or more should have VTE risk reduction strategies employed, such as limiting operating room times, weight reduction, discontinuation of hormone replacement therapy, and early postoperative mobilization. Patients undergoing major plastic and reconstructive operative procedures performed during general anesthesia that last longer than 60 minutes should receive VTE prevention. For patients with Caprini score of 3 to 6, the use of postoperative low-molecular-weight heparin (LMWH) or unfractionated heparin (UH) should be considered. For patients with Caprini score of 3 or more, use of mechanical prophylaxis throughout the duration of chemical prophylaxis for non-ambulatory patients should be considered. For patients with Caprini score of 7 or more, the use of extended LMWH postoperative prophylaxis should be strongly considered.

Aspirin does not decrease the risk of VTE and may increase the risk of perioperative complications.

2015

How well did you know this?
1
Not at all
2
3
4
5
Perfectly
39
Q

A 55-year-old woman comes to the office for a second opinion because she is displeased with the results of a recent bilateral mastectomy and breast reconstruction with 800-mL high-profile silicone implants. A photograph is shown. BMI is 35 kg/m2. She repeatedly shows pictures of models with augmented breasts and says that she wants her breasts to be “perkier.” She requests augmentation/mastopexy. Which of the following is the most appropriate next step in management?

A) Augmentation/mastopexy
B) Implant exchange
C) Mastopexy
D) Reassurance
E) Referral to a psychiatrist

A

The correct response is Option D.

The most reasonable approach in this patient is to offer reassurance and reset her expectations. A patient with a BMI of 35 kg/m2 who undergoes mastectomy and implant reconstruction will never look like a model with augmented breasts. This patient clearly has misguided expectations. Any surgical intervention is unlikely to produce the result she is looking for, when in fact she has a very acceptable result as is. Referral of this patient to a psychiatrist will likely upset the patient and undermine her trust.

2015

How well did you know this?
1
Not at all
2
3
4
5
Perfectly
40
Q

A 42-year-old woman comes to the office for treatment after receiving a diagnosis of cancer of the right breast. She has decided to undergo mastectomy of the right breast. Which of the following is a relative CONTRAINDICATION to nipple-sparing mastectomy?

A) Comedo-type breast tumor
B) Invasive lobular carcinoma
C) Subareolar tumor
D) Tumor location 3 cm from the nipple
E) Tumor size of 2.5 cm

A

The correct response is Option C.

A relative contraindication to nipple-sparing mastectomy is a centrally located tumor. Although various authors have employed different distance criteria, it is generally accepted that patients whose tumors are within 2 cm of the nipple are not candidates for nipple-sparing mastectomy.

Nipple-sparing mastectomy is an appropriate option for high-risk patients undergoing prophylactic mastectomy and for patients diagnosed with breast cancer who meet certain criteria. Those criteria are: tumor size of 3 cm or less, at least 2 cm from the nipple, not multicentric, and with clinically negative nodes.

Comedo carcinoma of the breast is a type of ductal carcinoma in situ. It is considered to be an early stage of breast cancer, is confined to the ducts, and usually does not spread beyond. It is not a contraindication to nipple-sparing mastectomy.

Invasive lobular carcinoma originates from the breast lobules, may form a thickening of the breast tissue rather than a discrete mass, and is often bilateral. As long as it meets the above criteria, it is not a contraindication to nipple-sparing mastectomy.

Inflammatory breast cancer, Paget disease, and tumors infiltrating the skin are also not candidates for skin-sparing or nipple-sparing mastectomy, according to several authors.

In more recent studies, a tumor size of 3 cm or less appears to result in no increase in local or regional recurrence in nipple-sparing mastectomy compared with alternative surgical approaches. A tumor of 2.5 cm is not a contraindication to nipple-sparing mastectomy.

2015

How well did you know this?
1
Not at all
2
3
4
5
Perfectly
41
Q

Which of the following characteristics is correlated with increased risk of nipple-areola complex necrosis in nipple-sparing mastectomies with immediate reconstruction?

A) Autologous tissue reconstruction
B) Direct to implant reconstruction
C) Patient age
D) Periareolar incision
E) Small breast size

A

The correct response is Option D.

Nipple-sparing mastectomies (NSMs) are becoming more common for both therapeutic and prophylactic mastectomies. Nipple-areola complex (NAC) necrosis can imperil reconstructive efforts, as well as negatively affect patients emotionally. It is important to maximize perfusion to the mastectomy skin flaps and NAC while still performing an oncologically sound procedure. There are multiple different incisions for performing NSM. Periareolar, inframammary-fold, radial, and vertical incisions are the most common. Periareolar incisions are associated with an increased risk of NAC necrosis in NSMs. Type of reconstruction, small breast size, and patient age have not been shown to be linked to increased rates of NAC necrosis.

2015

How well did you know this?
1
Not at all
2
3
4
5
Perfectly
42
Q

A 52-year-old woman receives a diagnosis of invasive ductal carcinoma of the right breast. Which of the following details from this patient’s history is the strongest risk factor for this diagnosis?

A) Early first pregnancy (less than 30 years)
B) Early menarche (less than 12 years)
C) Early menopause (less than 55 years)
D) Multiple episodes of breast-feeding
E) Remote oral contraceptive use

A

The correct response is Option B.

Early menarche is the highest risk factor for breast cancer of the options listed. Late first pregnancy, late menopause, no breast-feeding, and recent oral contraceptive use are also risk factors for breast cancer but are not as high risk.

How well did you know this?
1
Not at all
2
3
4
5
Perfectly
43
Q

A 42-year-old woman with a 3-cm invasive ductal carcinoma of the right breast is evaluated for breast reconstruction. She has not decided how she wants to manage her contralateral breast. Regarding eliciting a family history, which of the following cancers is associated with a mutation in a breast cancer-susceptibility gene?

A) Colon
B) Esophageal
C) Lung
D) Pancreatic
E) Thyroid

A

The correct response is Option D.

The breast cancer-susceptibility gene types 1 and 2 (BRCA1 and BRCA2) are tumor suppressor genes. Mutations in BRCA1 and BRCA2 are associated with hereditary breast and ovarian cancers. Additionally, they can be associated with increased risks of pancreatic and prostate cancer. Thyroid, lung, esophageal, and colon cancer are not associated with increased risks of BRCA1 and BRCA2 mutations.

2015

How well did you know this?
1
Not at all
2
3
4
5
Perfectly
44
Q

A 35-year-old woman comes to the office for consultation regarding prophylactic mastectomy and breast reconstruction. The patient’s mother and sister were diagnosed with bilateral breast cancer in their premenopausal years. Genetic testing for BRCA mutations is negative. Which of the following best estimates this patient’s lifetime risk of breast cancer?

A) 5%
B) 13%
C) 20%
D) 45%
E) 80%

A

The correct response is Option D.

The cumulative lifetime risk for a 35-year-old woman whose mother and sister had breast cancer is estimated to be approximately 15%. The risk may increase to as high as 45% if those cancers were premenopausal and bilateral. BRCA hereditary cancer is characterized by autosomal dominant genetics with multiple family members in each generation being affected. For patients with BRCA1 mutation, the risk of breast cancer has been estimated to be between 50 and 80% by age 65 years. The risk of developing ovarian cancer has been estimated to be 10% by age 60 years.

2015

How well did you know this?
1
Not at all
2
3
4
5
Perfectly
45
Q

A 39-year-old woman with a history of fibrocystic breast lesions comes to the office for consultation. She has no family history of breast cancer. Results of routine mammograms have been negative; she has never undergone biopsy. Which of the following is the most appropriate recommendation for this patient regarding managing her risk of breast cancer?

A) Continue to schedule routine mammograms
B) Refer for mastectomy
C) Schedule core needle biopsy
D) Schedule fine-needle aspiration
E) Schedule genetic testing

A

The correct response is Option A.

Studies have shown the fibrocystic changes alone in the breast are not directly linked to an increased risk of breast cancer, so there is no indication for fine-needle aspiration or core biopsy. Cancer risk increases in benign breast disease with increased proliferation and atypical hyperplasia. Even with no family history of breast cancer, it is recommended that the patient continue routine mammograms. Fibrocystic breast disease has not been linked in the literature to an increased risk of mutations of the BRCA genes; therefore, genetic testing is unnecessary.

2015

How well did you know this?
1
Not at all
2
3
4
5
Perfectly
46
Q

An otherwise healthy 37-year-old woman presents for delayed microsurgical breast reconstruction. Which of the following is associated with use of tamoxifen?

A) Hemodynamic instability
B) Impaired wound healing
C) Increased bleeding
D) Seroma formation
E) Thromboembolic events

A

The correct response is Option E.

Breast cancers that are estrogen receptor positive may be responsive to adjuvant chemotherapy with selective estrogen receptor modulators such as tamoxifen, which can reduce recurrence and mortality. Tamoxifen is associated with thromboembolic events, such as deep venous thrombosis and pulmonary embolism. This prothrombotic effect has been postulated to be secondary to the effect of tamoxifen on estrogen receptors that are abundant within vascular endothelium.

Tamoxifen has been shown to be associated with increased rates of total flap loss and decreased rates of flap salvage when taken within 28 days of microsurgical breast reconstruction, which represents two half-lives of the active metabolite of tamoxifen (N-desmethyl tamoxifen, t1/2=14 days). It has therefore been recommended that in patients undergoing microsurgical breast reconstruction, tamoxifen be held for at least 28 days preoperatively. Some authors have further advised holding the medication postoperatively in addition to preoperatively.

Tamoxifen is not associated with impaired wound healing, increased bleeding, hemodynamic instability, or seroma formation.

2014

How well did you know this?
1
Not at all
2
3
4
5
Perfectly
47
Q

An otherwise healthy 38-year-old woman undergoes prophylactic bilateral mastectomy and immediate reconstruction with deep inferior epigastric artery perforator (DIEP) free flaps. Intraoperatively, the left DIEP flap appears congested before the conclusion of the case. The left deep inferior epigastric artery and vein (DIEA and DIEV) were anastomosed to the proximal internal mammary vessels. The vascular pedicle is evaluated and each anastomosis appears patent and not kinked; however, the venous congestion persists. Which of the following is the most appropriate management?

A) Anastomose the superficial inferior epigastric vein to an internal mammary vessel perforator
B) Convert to left prosthetic reconstruction
C) Infuse tissue plasminogen activator (tPA) to the DIEA
D) Initiate leech therapy
E) Revise the DIEV anastomosis to the retrograde internal mammary vessel limb

A

The correct response is Option A.

Venous drainage of the lower abdominal skin and subcutaneous tissue occurs primarily through the superficial venous system and secondarily through the deep venous system, with perforating veins interconnecting the two systems. These communicating veins have been identified on computed tomography angiography in approximately 90% of abdominal walls in vivo. The majority of the remaining 10% of patients likely have communicating veins that are too small to visualize or are absent. In these cases of anatomical superficial venous system dominance, venous drainage is dependent on the superficial venous system. A recently published 2012 article by Sbitany et al. demonstrated that the incidence of intraoperative venous congestion secondary to persistent superficial venous system dominance was 0.9% in 1201 muscle-sparing transverse rectus abdominis musculocutaneous and deep inferior epigastric artery perforator free flaps. A free flap that becomes congested after reperfusion in the operating room should be assessed immediately for possible etiologies including twisting, kinking, tension, or vasospasm of the vascular pedicle. If a technical problem is ruled out and the venous anastomosis remains patent, obligatory enhancement of venous drainage with the superficial venous system is necessary to salvage the free flap rather than revision of the original anastamosis. Various methods include an anastomosis of the superficial inferior epigastric vein (SIEV) to the DIEV system or any chest wall vein, including the retrograde limb of the internal mammary vessel, the branch of the internal mammary vessel, or the thoracodorsal system. This requires preemptive planning and sparing of the superficial epigastric vein or SIEV during the dissection of the flap. A vein graft can be utilized if additional length is necessary. Another option is to substitute the DIEV anastomosis with the SIEV.

Tissue plasminogen activator (tPA) would not be indicated in this scenario, as it is used as a thrombolytic and there is no evidence of vascular thrombosis. Revising the DIEV anastomosis would be moot because it is patent and the deep system is being drained. Leech therapy is useful for venous congestion, but primarily as an adjunct after potential surgical etiologies have been addressed. Sacrificing the free flap without first attempting salvage is not warranted, and using a prosthetic would be possible only if prior patient consent were obtained.

2014

How well did you know this?
1
Not at all
2
3
4
5
Perfectly
48
Q

A 60-year-old woman with breast cancer undergoes a transverse rectus abdominis musculocutaneous flap breast reconstruction after mastectomy. She has no allergies. Weight is 200 lb (91 kg). Estimated blood loss is 200 mL. Duration of the operation is 3 hours and 50 minutes. Administration of cefazolin before skin incision is planned as prophylaxis against surgical site infection. Which of the following is the most appropriate dosage and timing of this injection?

DoseTimingRedosing

A) 1g, 5 minutes prior, after 150 mL of blood loss

B) 1g, 15 minutes prior, no

C) 1g, 40 minutes prior, no

D) 2g, 15 minutes prior, no

E) 2g, 40 minutes prior, no

A

The correct response is Option E.

There has been a renewed interest in perioperative antibiotics in recent years toward more appropriate use to decrease surgical site infection (SSI) while decreasing the incidence of resistant bacteria.

Current recommendations are to administer a single perioperative dose of antibiotics against common skin flora (gram positive), usually using a first-generation cephalosporin. However, the following recommendations may be underappreciated:

Cefazolin intravenous:1 g if <80kg; 2g if >80kg

Alternatives:clindamycin 600 to 900mg intravenously;
vancomycin 1 to 1.5g intravenously

In this case, the patient weighs 200lb (91kg), so 2g of cefazolin is the recommended dosage.

Additionally, an important factor is the timing of the administration of the perioperative antibiotics in order to achieve proper skin levels before incision. In one study by Classen et al., published in the New England Journal of Medicine in 1992, the optimal time was between 2 hours before the operation and skin incision, as greater than 2 hours before and any time after skin incision led to marked increases in the relative risk of SSIs. A follow-up by Weber et al. in 2008 narrowed the most appropriate window to between 30 and 59 minutes before skin incision.

Finally, there is the issue of redosing. Current recommendations are to redose if there is excessive blood loss (>1500 mL) or if there are long procedures where one exceeds the half-life of the antibiotic used.

In the clinical scenario described, the most appropriate choice is 2 g of cefazolin, because the patient’s weight is above 80 kg, administered between 30 to 59 minutes before skin incision. Redosing on the basis of blood loss is unnecessary, although one could consider redosing at approximately 4 hours on the basis of half-life.

2014

How well did you know this?
1
Not at all
2
3
4
5
Perfectly
49
Q

A 42-year-old woman who underwent mastectomy of the right breast 6 months ago is evaluated for delayed autologous breast reconstruction with free tissue transfer. Which of the following medications should be discontinued preoperatively if she is routinely taking it?

A) Diltiazem
B) Fluoxetine
C) Metoprolol
D) Multivitamin
E) Tamoxifen

A

The correct response is Option E.

It is well known that tamoxifen can increase the risk of thrombembolic events. In a retrospective study at MD Anderson Cancer Center, it was shown that patients who received the drug close to the procedure had a significantly higher rate of complications. It is recommended that the patient stop tamoxifen at least 28 days before surgery.

Other listed medications do not have a direct effect on thrombotic complications.

2014

50
Q

When performing immediate breast reconstruction, it is important to reconstruct the lateral inframammary fold. This is because the oncologic extirpation of the breast is carried out to which of the following anatomic landmarks?

A) Anterior edge of the latissimus dorsi muscle
B) Anterior edge of the serratus muscle
C) Lateral edge of the pectoralis major muscle
D) Lateral edge of the pectoralis minor muscle
E) Posterior edge of the serratus muscle

A

The correct response is Option A.

For modified radical and simple mastectomies, the landmarks of dissection are: superiorly to the clavicle, medially to the sternum, inferiorly to the inframammary fold, and laterally to the border of the latissimus dorsi muscle. The pectoralis major muscle fascia is resected with the specimen.

The recreation of the inframammary fold is important for shaping in breast reconstruction and care must be taken to evaluate and repair both the inferior and lateral components of the inframammary fold.

2014

51
Q

A 52-year-old woman is evaluated for breast reconstruction after modified radical mastectomy with adjuvant chemotherapy and radiation therapy 18 months ago. Her last radiation treatment was 8 months ago. BMI is 29 kg/m2. Examination today shows hyperpigmentation of the right chest wall with no redundancy of the mastectomy skin flaps. Her contralateral breast is a D cup with grade III ptosis. Which of the following methods will create the best symmetry for this patient?

A) Deep inferior epigastric perforator flap
B) Gel breast implant and acellular dermal matrix
C) Latissimus dorsi musculocutaneous flap
D) Tissue expander and acellular dermal matrix placement with planned staged exchange for permanent gel implant
E) Tissue expander placement alone with planned staged exchange for permanent gel implant

A

The correct response is Option A.

The deep inferior epigastric perforator flap would give the patient autologous tissue reconstruction with ample tissue for skin resurfacing and soft tissue for volume. In this radiated patient with a tight skin envelope, a tissue expander/implant, with or without acellular dermal matrix, would be difficult to create an appropriately ample skin envelope and the patient would be at higher risk for wound-healing problems and capsular contracture. The latissimus dorsi musculocutaneous flap, although an autologous tissue reconstruction, would have insufficient volume to adequately match this patient’s contralateral side. It would have to be combined with an implant.

2014

52
Q

A 19-year-old woman with a medical history significant for Poland syndrome and a BMI of 19 kg/m2 undergoes first-stage breast reconstruction with a tissue expander that is complicated by extrusion and infection 40 days after implantation. A photograph is shown. Attempts at implant salvage are made. The presence of which of the following factors is most likely to lead to decreased salvage rates?

A) BMI of 19 kg/m2
B) Culture-positive Staphylococcus sp
C) Hemoglobin A1c of 6.5%
D) Prepectoral placement of the device
E) Use of acellular dermal matrix

A

The correct response is Option B.

It has been shown that successful breast device salvage in breast reconstruction is possible if caught early. However, there are associated factors with failure, including culture-positive Staphylococcus (epidermidis or aureus), as demonstrated by several studies. Other associated risk factors for failure include obesity, poorly-controlled diabetes, smoking, history of radiation therapy, postoperative seroma, and early contamination of the implant with biofilm formation. Therefore, prompt and aggressive intervention is warranted in these situations where the device is threatened by either infection and/or exposure. This includes both surgical and antimicrobial options.

In a 2017 study, prepectoral and subpectoral placement demonstrated comparable complications. Acellular dermal matrix did not increase failure rates.

2019

53
Q

A 42-year-old woman with a history of a cesarean delivery from a low-transverse abdominal incision is scheduled to undergo a unilateral deep inferior epigastric perforator (DIEP) flap breast reconstruction. BMI is 28 kg/m² and the distance from nipple to sternal notch is 24 cm per side. This patient’s history of cesarean delivery is most likely to have which of the following effects?

A) Decreased abdominal seroma
B) Decreased flap venous congestion
C) Increased flap arterial thrombosis
D) Increased flap fat necrosis
E) No overall effect

A

The correct response is Option B.

Pfannenstiel incisions are the preferred access for cesarean deliveries. They are not a contraindication for abdominal-based flaps for breast reconstruction because the deep inferior epigastric circulation is not disturbed. However, the superficial epigastric circulation may be divided. The net result appears to be a more robust venous circulation with a protective effect against fat necrosis in the flap. This venous division causes a delay-type phenomenon—during healing increased branches are formed from the superficial epigastric circulation, and there is some evidence of new connections to the venae comitantes of the deep epigastric venous circulation.

There is evidence, however, for an increased rate of abdominal healing problems, including seroma (15% versus 6%), wound healing problems, and fat necrosis in the abdomen. There is no evidence for an effect on the arterial circulation of the flap.

2019

54
Q

A 50-year-old woman comes to the office 6 weeks after undergoing right mastectomy and immediate placement of a tissue expander. She reports swelling and redness of the right breast. A photograph is shown. Which of the following factors is most predictive of implant salvage failure in this patient?

A) Culture positive for Pseudomonas species
B) Elevated body mass index
C) Periprosthetic seroma
D) Presence of cellulitis
E) Previous irradiation

A

The correct response is Option A.

Immediate implant-based reconstruction has become increasingly popular over the past two decades, accounting for over 70% of all reconstructions in the United States. The benefits of immediate reconstruction are numerous, including decreased recovery/number of required procedures and increased patient psychological well-being and aesthetic outcome. However, the complication (seroma, mastectomy flap necrosis, loss of implant, and infection) rates after implant-based reconstruction remain relatively high. Infection rates in the reported literature range from 2.5 to 24%.

Historically, periprosthetic infection or implant exposure mandated immediate implant removal. However, numerous studies over the past several decades have demonstrated implant salvage rates of 37.3 to 73% depending on the methods employed. Several studies have looked at the predictive factors that increase the risk of a failed salvage attempt. Salvage was typically defined as administration of systemic antibiotics (oral or intravenous), removal of the infected implant, partial/total capsulectomy, pocket curettage, implant pocket irrigation with antibiotic solution, and placement of a new device.

Factors associated with implant salvage failure include an elevated white blood cell count, elevated temperature, deep-seated pocket infection (purulent periprosthetic fluid), and atypical pathogens such as methicillin-resistant Staphylococcus aureus (MRSA) and Pseudomonas species. Spear et al. showed that 93.9% of mild implant infections (localized cellulitis) could be salvaged compared with a 30% salvage rate in the severe infection group. Factors such as smoking, chemotherapy, previous irradiation, mastectomy skin necrosis, increased BMI, and use of acellular dermal matrix (ADM) have demonstrated increased rates of implant-related infections, but these factors have not been demonstrated to increase the risk of implant salvage failure.

2019

55
Q

A 68-year-old woman comes to the office for a delayed breast reconstruction. She had right breast cancer and a mastectomy followed by chemotherapy and radiation therapy 1 year ago. BMI is 35 kg/m2. Medical history includes well-controlled type 2 diabetes mellitus, a previous cesarean section through a low transverse incision, and an open cholecystectomy through a subcostal incision. The patient requests autologous reconstruction, but the surgeon is not comfortable performing a free flap. Which of the following is the most appropriate method for reconstruction?

A) Bipedicled transverse rectus abdominis musculocutaneous (TRAM) flap
B) Contralateral pedicled TRAM flap
C) Ipsilateral pedicled TRAM flap
D) Surgical delay procedure followed by contralateral pedicled TRAM flap
E) Surgical delay procedure followed by ipsilateral pedicled TRAM flap

A

The correct response is Option D.

In this obese patient with right breast cancer and a previous subcostal incision, a delay procedure with a contralateral transverse rectus abdominis musculocutaneous (TRAM) flap is the most appropriate method for reconstruction. Although a contralateral TRAM flap can be performed without a delay procedure, it has been shown that the addition of a delay procedure decreases risks of ischemia to the flap. In addition, a delay procedure may also reduce risks of abdominal wall complications. In this patient, the subcostal incision excludes right-sided pedicled flap reconstruction, so an ipsilateral pedicled TRAM would not be the right choice, nor would a bipedicled TRAM flap.

Although there is controversy on which patients should have a delay procedure, the use of this technique has usually been limited to high-risk patients and to those requiring large amounts of tissue. Multiple reports have shown that obese patients undergoing a delay can decrease the risks of tissue related ischemia. In a paper by Wang et al., the delay procedure was performed at least 14 days prior to the reconstruction; however, other studies have shown improvements at 7 days. The procedure described consists of ligation of both deep inferior epigastric arteries and veins bilaterally accessed from an inferior flap incision. This can also be done laparoscopically. Some authors advocate more extensive incisions and elevating portions of the flap; however, there is little data to show that this is effective or necessary.

2019

56
Q

A 52-year-old woman undergoes autologous breast reconstruction with unilateral deep inferior epigastric perforator (DIEP) flaps. According to the Hartrampf model of perfusion zones, if the lateral row perforator vessels are used, in which chronological order will the flap zones be perfused?

A) I – II – III – IV
B) I – III – II – IV
C) II – I – III – IV
D) II – I – IV – III
E) IV – III – II – I

A

The correct response is Option B.

In medial perforator-based flaps, the zones are perfused in the order I – II – III – IV (A) as shown in the image. In lateral perforator-based flaps, however, the zones are perfused in the order I – III – II – IV (B).

57
Q

A 64-year-old woman who is postmenopausal asks why she has not been prescribed hormone replacement therapy with estrogen and progestin like her mother was. Supplementation with these hormones is associated with an increased risk for which of the following?

A) Coronary artery disease
B) Diabetes
C) Endometrial cancer
D) Invasive breast cancer
E) Osteoporosis

A

The correct response is Option D.

Hormone replacement therapy has fallen out of favor because of a risk profile that is believed to exceed the potential benefits. Combined estrogen and progestin supplementation is thought to be associated with an increased risk for invasive breast cancer but may decrease the risk for diabetes and osteoporosis. It is thought to not impact the risk for coronary artery disease or endometrial cancer.

2019

58
Q

A 46-year-old woman who is 5 ft 7 in (170 cm) tall and weighs 135 lbs (61 kg) is evaluated one year following bilateral nipple-sparing mastectomy and immediate reconstruction with placement of 350-mL smooth, round silicone gel implants beneath the pectoralis major muscle. Since the surgery, she has experienced hyperdynamic deformity of her breasts. On physical examination, the breast reconstruction appears natural, and there is significant movement of the breasts when the patient flexes her chest. Which of the following is the most appropriate management for this patient?

A) Inject botulinum toxin into the pectoralis major muscle
B) Inject triamcinolone-40 into the areas of tenderness using ultrasound guidance
C) Move the implants to the prepectoral plane and cover them fully with acellular dermal matrix
D) Perform a breast MRI to assess for rupture of the implants
E) Refer the patient to a physical therapist for range of motion, massage, and ultrasound treatments

A

The correct response is Option C.

This patient is experiencing significant movement because her implants were placed beneath the pectoralis major muscles. While reconstruction options are limited in this otherwise healthy and very thin patient who is not a good candidate for fat grafting or pedicled or free tissue transfer, placing implants over the pectoralis major muscles and covering the implants fully with acellular dermal matrix would be the most appropriate method of reconstructing her breasts and addressing her concerns.

Physical therapy and muscle relaxants are unlikely to produce long-term improvement. An MRI would likely be nondiagnostic, and even if her implants were ruptured, change to a prepectoral plane is still indicated. Botulinum toxin type A is likely not as effective for long-term significant improvement as reoperation. Triamcinolone would not be effective for hyperdynamic deformity.

2019

59
Q

The capsules from patients with breast implant–associated anaplastic large-cell lymphoma (ALCL) have significant presence of which of the following bacteria?

A) Escherichia coli
B) Ralstonia pickettii
C) Staphylococcus aureus
D) Pseudomonas aeruginosa
E) Serratia marcescens

A

The correct response is Option B.

Most concerning in the past two decades is the incidence of breast implant–associated anaplastic large-cell lymphoma (ALCL). This entity was first diagnosed and associated with breast implants in 1997, and is almost only associated with a history of textured implants and/or tissue expanders. The most common presentation of these patients is late seroma, with some patients presenting with mass, tumor erosion, or lymph node metastasis. A recent review of the world literature on this entity include the following: (1) 173 cases were documented, (2) no cases were found in patients with documented smooth devices only (although this remains controversial, as the data in many cases are incomplete), (3) there may be an associated genetic predisposition as suggested for cutaneous T-cell lymphoma, and (4) the cause is likely multifactorial.

Bacterial biofilm is thought to be an inciting factor for the development of both breast-implant related ALCL and Non-Tumor related capsule contractures. The capsules from patients with tumor had significant presence of Gram-negative bacteria (Ralstonia species) compared to nontumor capsules (Staphylococcus species). Such data may support the bacterial induction model, as there are also other types of implant-associated lymphomas.

2019

60
Q

A 45-year-old woman who is obese is considering unilateral mastectomy and reconstruction of the left breast because of invasive ductal carcinoma. Which of the following patient characteristics is associated with the lowest risk for complications from a nipple-sparing mastectomy?

A) BMI of 41 kg/m2
B) Grade III ptosis of the breast
C) Nipple retraction
D) Tumor distance from nipple of 5 cm
E) Tumor size of 6 cm

A

The correct response is Option D.

Nipple-sparing mastectomy is increasing in popularity. To decrease the risk for surgical complications as well as oncologic complications, smaller tumors located further from the nipple in patients without morbid obesity or severe ptosis are considered better candidates for treatment with nipple-sparing mastectomy. Clinical involvement of the nipple, including retraction, would suggest that nipple-sparing mastectomy should not be performed.

2019

61
Q

Which of the following is the most likely chronic effect of post-mastectomy radiation therapy?

A) Desquamation
B) Dyspigmentation
C) Edema
D) Erythema
E) Ulceration

A

The correct response is Option B.

Radiation therapy induces tissue injury that can be categorized as acute or chronic. The spectrum of acute injury includes erythema, edema, desquamation, hyperpigmentation, and ulceration, ranging from mild to severe. Acute radiation dermatitis occurs in upward of 85% of treated patients. Chronic injury involves skin atrophy, dryness, telangiectasia, dyspigmentation, and dyschromia. In the breast, it leads to chronic fibrosis of the skin and subcutaneous tissues. This fibrosis and surrounding injury can lead to pain and restricted movement of the arm. The chronic changes from radiation can take months to years to fully manifest.

2019

62
Q

A 43-year-old woman who is BRCA-positive is scheduled to undergo bilateral mastectomy. Tissue expander–based reconstruction is planned. Which of the following is the optimal duration of antibiotic prophylaxis for this patient?

A) No preoperative antibiotic
B) One preoperative antibiotic dose and another dose during skin closure
C) One preoperative antibiotic dose, followed by 24 hours of treatment while in the hospital
D) One preoperative antibiotic dose, followed by 24 hours of treatment while in the hospital and then discharge on oral antibiotics until drains are removed
E) One preoperative antibiotic dose, followed by 24 hours of treatment while in the hospital and then maintenance on oral antibiotics until tissue expanders are exchanged

A

The correct response is Option C.

The overall complication rate in breast reconstructive surgery is as high as 60%. Infection rates can exceed 20%, much higher than in clean elective surgery. The CDC guidelines suggest only 24 hours of peri-operative antibiotics beginning thirty minutes prior to skin incision. However, not all plastic surgeons agree with this. A 2013 meta-analysis found when comparing combined patient cohorts receiving no antibiotics, antibiotics for less than 24 hours, and antibiotics for greater than 24 hours, the average infection rates were 14.4, 5.8, and 5.8%, respectively. This demonstrated that the administration of antibiotics made a difference, however duration beyond 24 hours did not.

A study was published in 2013 evaluating the difference in surgical site infection between two different prophylactic antibiotic durations (24 hours and until drain removal). In this prospective, randomized, controlled non-inferiority trial, 24 hours of antibiotics is equivalent to extended oral antibiotics for surgical-site infection in tissue expander immediate breast reconstruction patients.

2019

63
Q

A 45-year-old woman comes to the office for consultation about immediate bilateral breast reconstruction of a right-sided tumor measuring 2.5 cm. Biopsy reveals a HER-2/neu negative invasive ductal carcinoma without lymphovascular invasion. The patient requests nipple-sparing mastectomy. Physical examination shows a palpable mass is located in the right upper outer quadrant approximately 1.5 cm from the nipple-areola complex and is freely mobile. There is no lymphadenopathy on exam. Based on current literature, which of the following best describes this patient’s candidacy for the requested procedure?

A) Good candidate based on current presentation
B) Not a candidate because of lymph node status
C) Not a candidate because of tumor location
D) Not a candidate because of tumor pathology
E) Not a candidate because of tumor size

A

The correct response is Option C.

Nipple-sparing mastectomy or total skin-sparing mastectomy is becoming an increasingly popular choice for women because of the excellent cosmetic outcomes and the ability to save the nipple-areola complex that may provide psychological benefits with increased patient satisfaction as well. Nipple-sparing mastectomy appears to be oncologically safe with low risks of cancer recurrence in the literature thus far. However, there has been little long-term follow-up, so this approach is still somewhat controversial because the oncologic safety and locoregional recurrence have not been examined definitively. Although certain centers are pushing the envelope regarding the use of this technique in a wide range of patients, the current literature supports the following exclusion criteria:

A. Tumor size greater than 5 cm B. Tumor location less than 2 cm from the nipple C. Evidence of axillary disease D. Tumor involvement on retroareolar biopsy E. Lymphovascular invasion, human epidermal growth factor receptor-2 positivity, and/or HER-2/neu positivity on biopsy

The current patient’s tumor was found to have a tumor-to-nipple distance of 1.5 cm which is a relative contraindication to nipple-sparing mastectomy in this case.

2019

64
Q

A 56-year-old woman is evaluated 6 hours after undergoing bilateral breast reconstruction with a deep inferior epigastric perforator (DIEP) flap. Doppler examination shows strong arterial signals in both flaps. The right breast appears bluish with a capillary refill time of 1 second compared to 3 seconds on the left side. Which of the following is the most appropriate next step?

A) Administration of tissue plasminogen activator
B) Application of leeches
C) Application of nitroglycerin ointment
D) Exploration in the operating room
E) Observation

A

The correct response is Option D.

The patient described has evidence of venous congestion. The reported incidence of venous congestion in free tissue breast reconstruction ranges from 2 to 20%. Causes include venous thrombosis, inadequate perforator selection, and superficial venous system dominance with lack of sufficient communication to the deep system. Signs of venous compromise include the following: cyanotic/blue color, brisker than normal capillary refill, increased tissue turgor, cooler temperature compared to normal skin (greater than 2 degrees), rapid bleeding of dark blood with pinprick, and absence of continuous venous Doppler signal. The most appropriate course of action in this scenario is emergent exploration in the operating room to assess the vascular pedicle for thrombosis, compression from hematoma, kinking, or superficial system dominance. Flap salvage rate is directly tied to timing of exploration, with higher salvage rates in flaps explored within 6 hours of identification of compromise.

Early recognition and rapid exploration of compromised flaps are the most important factors predicting flap salvage, so observation would be unacceptable. Tissue plasminogen activator is useful if diffuse clotting is suspected within the flap, but should only be given locally within the flap. Leeches can be a useful adjunct postoperatively after employing the other maneuvers described above, but would not resolve the underlying problem in this case. Application of topical nitroglycerin can improve venous congestion in random skin flaps, but has no role in the management of acute microvascular thrombosis.

2020

65
Q

A 45-year-old woman presents with right breast cancer and is planning a nipple-sparing mastectomy and tissue expander placement. She is specifically interested in a carbon dioxide–based expander. Which of the following is a disadvantage of this device compared with a saline tissue expander?

A) Extrusion
B) Inability to deflate
C) Increase in wound dehiscence
D) Increase in wound infection
E) Possible device dislocation

A

The correct response is Option B.

The carbon dioxide-based tissue expander (AeroForm) is a fixed-volume device and has an inability to deflate the expander.

In a prospective, multicenter, randomized controlled trial comparing carbon dioxide–based expanders and saline tissue expanders, there were no statistically significant differences in rates of wound infection, extrusion, device dislocation, or wound dehiscence. Advantages of the carbon dioxide–based expander include a more rapid expansion process and a shorter time to implant exchange. The device is self-contained and patient-controlled, so there are no needles required and possibly fewer physician office visits.

2020

66
Q

A 35-year-old woman presents with unilateral swelling that has developed over the past 3 months. She underwent bilateral nipple-sparing mastectomy with immediate implant reconstruction with textured, round silicone gel implants 8 years ago. Ultrasound confirms periprosthetic seroma without any masses. Which of the following is the most appropriate next step in the management of this patient?

A) Core needle biopsy
B) Fine-needle aspiration
C) Implant removal and capsulectomy
D) MRI
E) Positron emission tomography (PET) scan

A

The correct response is Option B.

The clinical scenario is concerning for breast implant–associated anaplastic large-cell lymphoma (BIA-ALCL). Aspiration of the fluid seen on ultrasonound and pathologic evaluation is necessary to confirm the diagnosis. Following the National Comprehensive Cancer Network guidelines, initial workup of an enlarged breast should include ultrasound evaluation specifically for a fluid collection, a breast mass, or enlarged regional lymph nodes (axillary, supraclavicular, and internal mammary).

MRI is appropriate for cases where ultrasound is indeterminate or requires further confirmation. This patient does not have an identifiable mass amenable to core biopsy. Positron emission tomography (PET) scan is beneficial in confirmed cases to identify associated masses, chest wall involvement, regional lymphadenopathy, and/or metastasis. Implant removal and capsulectomy is appropriate once the diagnosis is confirmed.

2020

67
Q

In women undergoing prosthetic breast reconstruction complicated by an expander/implant infection, which of the following is the most common gram-negative bacteria isolated from cultures?

A) Escherichia coli
B) Klebsiella
C) Proteus
D) Pseudomonas
E) Serratia

A

The correct response is Option D.

Tissue expander/implant-based breast reconstruction remains the most common form of reconstruction after mastectomy. One of the most potentially devastating complications of this form of breast reconstruction is an implant infection with need for removal of the expander/implant. The mean reported incidence of implant infection after breast reconstruction is 8%, with a range of 1 to 35%. When cultures are obtained, the most common causative bacteria on microbiology examination are gram-positive organisms (41 to 83%), specifically, Staphylococcus species (56 to 76.5%). Gram-negative bacteria accounted for 15.3 to 28.6%, with Pseudomonas (10.7 to 14%) being the most common gram-negative bacteria present on microbiology examination.

2020

68
Q

A 50-year-old woman presents to the clinic to discuss breast reconstruction after bilateral mastectomy. She is interested in free tissue transfer. She has a diagnosis of systemic lupus erythematosus treated with chronic steroid therapy and wants to know if she is an appropriate candidate for free flap reconstruction. Which of the following statements best describes the surgical risks for this patient with lupus compared with the general population?

A) Higher rate of free flap failure
B) Higher risk of a thromboembolic event
C) Similar rate of hernias after abdominally based free flaps
D) Similar rates of infection

A

The correct response is Option B.

The statement which best describes the surgical risks for a patient with lupus undergoing free tissue transfer for breast reconstruction is that the patient has a higher risk of a thromboembolic event than the average patient.

The rate of free flap failure in patients with lupus is similar to patients without lupus. Chronic steroid use increases the risk of wound healing complications in patients with lupus, rather than increases the risk of free flap failure. Additionally, patients with lupus have an increased risk of abdominal wall bulge and hernia after abdominally based free flaps compared with the average population. Chronic steroid use also suppresses the immune system, predisposing patients treated with steroids to increased rates of infection compared to patients not taking steroids.

2020

69
Q

A 53-year-old woman is evaluated for left-sided nipple reconstruction after mastectomy. She has scars on the left breast from a previous breast biopsy, as well as from the mastectomy itself. Nipple reconstruction must be designed around the scars. In single-pedicle nipple reconstruction, which of the following provides the blood supply to the pedicle?

A) Internal mammary artery perforators
B) Posterior intercostal arteries
C) Subdermal plexus
D) Superior intercostal artery
E) Thoracoacromial artery perforators

A

The correct response is Option C.

Single-pedicle nipple reconstructions, which include such techniques as the skate flap, star flap, C-V flap, and opposing tab flaps as well as other variations, create nipples from remaining mastectomy skin through adjacent tissue transfer. The flap derives its blood supply from the subdermal plexus.

The creation of the flap must keep this blood supply in mind. The flap design must avoid previous scars at the flap base and must integrate the subcutaneous fat at the base of the pedicle.

The internal mammary artery supplies the breast itself and the nipple-areola complex, and the thoracoacromial artery supplies the pectoralis muscle and the breast. The posterior intercostal arteries supply the intercostal spaces. The superior intercostal artery arises from the costocervical trunk, off of the subclavian artery, and supplies the intercostal spaces.

2020

70
Q

A 44-year-old previously healthy woman comes to the clinic because of a 2-week history of a painless mass in the left breast. She initially felt this mass while taking a shower. Her mother was diagnosed with fibrocystic changes. The patient denies alcohol consumption and smoking cigarettes. Examination of the left breast shows a 5-cm mobile, painless mass in the left upper external quadrant without nipple discharge, skin retractions, or color changes. Examination of a specimen obtained on biopsy discloses a phyllodes tumor, and surgical excision of the lesion is planned. Which of the following is the most important factor to prevent local recurrence after surgery?

A) Adjuvant radiotherapy
B) Concurrent axillary node dissection
C) Postoperative chemotherapy
D) Surgical margins less than or equal to 0.5 cm
E) Wide surgical margins

A

The correct response is Option E.

In a young woman who has no history of breast cancer, presents with a painless mass, and has a mammogram suggestive of fibroadenoma but a core needle biopsy showing stromal hypercellularity with atypical spindle cells and a high mitotic rate, a phyllodes tumor must be suspected.

Phyllodes tumors are uncommon fibroepithelial breast tumors that behave like benign fibroadenomas, although they have a high propensity to recur locally. More aggressive tumors can metastasize distantly. Surgery is the preferred treatment for this condition. In this context, surgical margins greater than or equal to 1 cm have been associated with a lower recurrence rate in borderline and malignant tumors.

Axillary lymph node involvement is rare. Wide local excision or mastectomy with appropriate margins is the preferred clinical intervention.

Based on limited data, the role of systemic chemotherapy in phyllodes tumors is limited. Patients with benign or borderline phyllodes tumors are usually cured with surgery and should not be offered chemotherapy unless they develop unresectable metastases.

Local recurrence rate is higher after excision with narrower margins than broader ones. The efficacy of postoperative adjuvant radiotherapy for a breast phyllodes tumor is not clear. In clinical practice, the utilization of adjuvant radiotherapy for a phyllodes tumor appears to be modest.

2020

71
Q

A 54-year-old woman is evaluated for nipple-areola complex reconstruction after mastectomy and silicone implant-based reconstruction. During discussion of the risks and benefits of a C-V flap, the patient asks about the long-term results of different techniques. Which of the following is the most likely long-term complication of a single-pedicle nipple-areola reconstruction?

A) Atrophic scarring
B) Delayed nipple necrosis
C) Hypertrophic scarring
D) Implant exposure
E) Loss of projection

A

The correct response is Option E.

Single-pedicle nipple reconstructions, which include techniques such as the skate flap, star flap, C-V flap, and opposing tab flaps as well as other variations, create nipples from remaining mastectomy skin through adjacent tissue transfer. The flaps derive their random-pattern blood supply from the subdermal plexus.

The creation of the flap must keep this blood supply in mind. The surgical technique must avoid previous scars at the base of the flap design and must integrate the subcutaneous fat at the base of the pedicle.

While hypertrophic and atrophic scarring can occur, they are not the most common long-term effects, and are more a function of patient characteristics than flap characteristics.

Implant exposure can occur with scar breakdown, but this is an early rather than a late complication.

Delayed nipple necrosis is technically not correct because the nipple is no longer present, and is not correct of the nipple reconstruction because necrosis of the flaps, if it occurs, usually occurs early.

2020

72
Q

A patient with a history of breast cancer undergoes nipple-sparing mastectomy of the right breast with immediate implant-based reconstruction. Ten months after surgery, the patient starts to recover sensitivity at the nipple. Which of the following nerves is most likely providing sensitivity to the nipple-areola complex in this patient?

A) Anterior branch of the fourth intercostal nerve
B) Anterior branch of the second intercostal nerve
C) Lateral branch of the fifth intercostal nerve
D) Lateral branch of the fourth intercostal nerve
E) Lateral branch of the second intercostal nerve
F) Lateral branch of the third intercostal nerve

A

The correct response is Option A.

The nipple and areola of the breast are innervated by both the anterior and lateral cutaneous branches of the third, fourth, or fifth intercostal nerves. The anterior and lateral cutaneous branches of the second and sixth intercostal nerves innervate breast skin only.

In anatomical studies conducted in female cadavers, the fourth intercostal nerve’s lateral cutaneous branch supplied the nipples in 93% dissected breasts. The third and fifth intercostal lateral branches were found to innervate the nipple alone in 3.6%. However, the fourth intercostal lateral branch penetrates the deep fascia in the midaxillary line, takes an inferomedial course to reach the midclavicular line, and continues through the glandular tissue towards the posterior surface of the nipple. Thus, when a mastectomy is performed, this lateral branch is the most likely one to be dissected.

On the other hand, the anterior cutaneous branches take a superficial course, as they run in the subcutaneous tissue close to the skin and reach the nipple from the lateral side. According to this, the anterior branch of the fourth intercostal nerve is most likely providing sensitivity to the nipple-areolar complex after nipple-sparing mastectomy.

2020

73
Q

A 55-year-old woman with a BRCA gene mutation elects to undergo bilateral mastectomy with reconstruction using a deep inferior epigastric perforator flap. BMI is 41 kg/m2. Physical examination shows both supra- and infraumbilical adiposity with excess skin and a mature cesarean delivery scar. This patient has the highest risk for which of the following early postoperative complications?

A) Abdominal wall bulge
B) Abdominal wall hernia
C) Delayed wound healing of donor site
D) Fat necrosis of the flap
E) Flap failure

A

The correct response is Option C.

The highest risk is for delayed wound healing of the donor site. Because of the patient’s morbid obesity and prior cesarean delivery scar, she has the highest risk for some form of wound breakdown or prolonged wound healing. This risk can be as high as 50 to 60% for morbidly obese patients. These trends are similar in patients following reconstruction with a transverse rectus abdominis musculocutaneous (TRAM) flap.

To reduce these risks, minimal undermining is recommended and only done if necessary. Techniques to preserve all cutaneous perforators will help reduce the risk associated with closure of the donor sites.

While morbid obesity can be associated with increased abdominal wall thickness, there is no correlation with the occurrence of abdominal wall bulge or hernia. The risks for these complications are less than 2%.

Patients with morbid obesity can have shorter operative times, but there is no correlation with overall flap failure, with rates reported to be less than 1%. This is also seen in pedicled and free TRAM flap reconstructions.

Rates of fat necrosis of the flap can be as high as 10 to 15% in patients undergoing reconstruction, but this risk is not affected by body habitus or body mass index and is lower than the risk for delayed wound healing.

2020

74
Q

A healthy 45-year-old woman with a history of breast malignancy underwent bilateral mastectomy and reconstruction with tissue expanders followed by exchange for cohesive silicone gel implants eight years ago with routine postoperative MRI surveillance. She comes to the office to report pain and tightness in the right breast that has gradually increased over the past month. On examination, temperature is 36.8°C (98.2°F), blood pressure is 112/76 mmHg, and heart rate is 68 bpm. The right breast appears fuller than the left breast; otherwise, the right implant is in a symmetric position with the left side. The skin is otherwise normal in appearance, and there is no tenderness on palpation. Which of the following is the most appropriate next step in management?

A) MRI of the right breast to assess the integrity of the implant
B) One week of an oral antibiotic and prednisone taper
C) Operative exploration, culture, and replacement of implant
D) Referral of the patient back to her medical and surgical oncologists
E) Ultrasound of the right breast and fine-needle aspiration of any fluid

A

The correct response is Option E.

Patients that present with a late seroma should be evaluated for possible Breast implant associated Anaplastic Large Cell Lymphoma (BI-ALCL). A late seroma is usually accepted as occurring 1 year following surgery: however there are cases of BI-ALCL seroma that have presented as early as 4 months.

The first step in evaluation for BI-ALCL is an ultrasound followed by fine needle aspiration is indicated. The fluid requires evaluation beyond routine cell cytology. Immunohistochemistry test for CD30 was the most commonly positive marker for BI-ALCL. Immunohistochemistry stains specific antigens in cells by binding to this antigen in an antibody/antigen reaction. The specific stain can then be seen under light microscopy. CD30 antibody labels anaplastic large cell lymphoma cells. CD30 is a transmembrane cytokine receptor belonging to the tumor necrosis factor receptor family and characteristically stains ALCL cells.

MRI for implant integrity and referral to her Oncologist may be needed but it is not the most appropriate next step. BIA-ALCL needs to be ruled out. Immediate operative exploration is not indicated before fluid aspiration and immunohistochemistry evaluation. Antibiotics and prednisone is not indicated in this patient without evidence of infection or inflammation (red breast syndrome).

2020

75
Q

An otherwise healthy 54-year-old perimenopausal woman is scheduled for a mastectomy for biopsy-proven right-sided grade 2 ductal carcinoma. According to the National Comprehensive Cancer Network (NCCN) guidelines, postmastectomy radiation therapy will be the standard of care for this patient if she has which of the following surgical outcomes?

A) 1-cm surgical margins, four positive axillary lymph nodes
B) 1-cm surgical margins, one positive axillary sentinel node
C) 1-mm surgical margins, no positive axillary nodes
D) 5-mm surgical margins, no positive axillary nodes
E) 5-mm surgical margins, three positive axillary nodes

A

The correct response is Option A.

Traditionally, the need for radiation therapy has been a contraindication for implant-based reconstruction, and autologous reconstruction is the conservative gold standard for women with advanced cancer needing postmastectomy radiation. More recently, there have been reports of successful implant based reconstruction in the setting of postmastectomy radiation that have similar complication profiles and good oncologic outcomes compared with autologous reconstruction. Protocols vary between those that radiate the expander and then expand, and those that expand and then radiate the permanent implant. Being able to anticipate which patient will require postmastectomy radiation is essential for joint decision making about breast reconstruction with the patient prior to her mastectomy.

By National Comprehensive Cancer Network (NCCN) guidelines, relative indications for postmastectomy radiation therapy include: positive sentinel node with unknown status of other axillary nodes, one to three positive nodes on permanent histology, and close surgical margins (less than 5 mm). Postmastectomy radiation is recommended as the standard of care in the situations of positive surgical margins with the inability to get clear margins and four or more positive lymph nodes.

2020

76
Q

A 44-year-old woman presents in evaluation for breast reconstruction with biopsy-proven left breast-infiltrating ductal carcinoma after routine mammography discovered a 7-cm lesion. She has been referred to medical oncology and genetic testing is pending. Her past medical history is significant for hypertension and scleroderma. On examination, she has grade I ptosis and wears a size 34A brassiere. During the consultation, the patient reports a strong preference for lumpectomy and oncoplastic reconstruction over total mastectomy. Which of the following is most likely to increase this patient’s chances of qualifying for breast-conserving therapy?

A) Active scleroderma
B) BRCA-1 gene mutation
C) Multicentric tumor
D) Preoperative chemotherapy
E) Small-sized breasts

A

The correct response is Option D.

Preoperative chemotherapy could increase this patient’s chances of qualifying for locoregional treatment (partial mastectomy or lumpectomy). Studies have shown that breast conservation rates are improved with preoperative systemic therapy, which can also render inoperable tumors resectable. Other potential benefits of preoperative (neoadjuvant) chemotherapy include providing important prognostic information based on response to therapy, minimizing the extent of axillary surgery, and allowing time for genetic testing and reconstructive planning prior to surgery. A small-sized breast would likely provide insufficient uninvolved breast tissue for breast-conserving therapy after resection of a large (7 cm) mass. The same applies to multicentric tumors.

Whole breast irradiation is strongly recommended after lumpectomy, with studies showing a favorable effect in reducing the 10-year risk of recurrence (19% versus 35%) and the 15-year risk of breast cancer death (21% versus 25%). Therefore, patients with (relative) contraindications to radiation therapy, such as lupus or scleroderma (connective tissue disease involving the skin), should ordinarily be offered total mastectomy, particularly if this resolves the need for radiation therapy. While radiation therapy would likely still be considered for this particular patient even after total mastectomy (tumor size greater than 5 cm), the diagnosis of scleroderma itself does not increase her chances of qualifying for breast conservation surgery. BRCA-1 gene mutation and other genetic predispositions to breast cancer are relative contraindications for breast-conserving therapy. These patients may be considered for prophylactic bilateral mastectomy for risk reduction.

2020

77
Q

A 41-year-old woman presents with right breast lobular carcinoma in situ (LCIS) involving a 1-cm area with no palpable axillary nodes. According to the TNM staging system, which of the following is this patient’s T classification?

A) Tx
B) Tis
C) T0
D) T1a
E) None; there is no TNM staging for LCIS

A

The correct response is Option E.

Lobular carcinoma in situ (LCIS) has been removed from the staging classification system in the 8th edition and is no longer included in the pathologic tumor in situ (pTis) category. LCIS is treated as a benign entity with an associated risk for developing carcinoma in the future but not as a malignancy capable of metastases. There is a small subset of LCIS that has high-grade nuclear features and may exhibit central necrosis. This subset has been referred to as pleomorphic LCIS and has histologic features that partially overlap the features of ductal carcinoma in situ (DCIS), including the potential to develop calcifications detectable by mammography. The expert panel debated whether to include this variant of LCIS in the pTis category; however, there are insufficient data in the literature regarding outcomes and reproducible diagnostic criteria for this LCIS variant. Cases exhibiting DCIS and LCIS are classified as pTis (DCIS).

2020

78
Q

An otherwise healthy, 45-year-old woman presents for breast reconstruction. She underwent a mastectomy 1 year ago with no immediate reconstruction followed by post-mastectomy radiation therapy. She does not want anything performed to the contralateral breast and does not want to have an implant. She has a history of an abdominoplasty. The plastic surgeon plans to perform stacked profunda artery perforator flaps for the unilateral reconstruction. Which of the following is the most appropriate option for the recipient vessels?

A) Ipsilateral and contralateral antegrade internal mammary vessels
B) Ipsilateral antegrade and retrograde internal mammary vessels
C) Ipsilateral antegrade and retrograde thoracodorsal vessels
D) Ipsilateral antegrade internal mammary and thoracoacromial vessels
E) Ipsilateral antegrade thoracodorsal and thoracoacromial vessels

A

The correct response is Option B.

It has been demonstrated that the retrograde intermammary vessels are a viable and reliable choice for stacked flap reconstruction. The benefit of using these as the recipient vessels is that it does not add another recipient site to the operation.

Ipsilateral and contralateral antegrade internal mammary vessels are incorrect because the patient does not want to have any procedures performed on her contralateral breast, although this has been described as an option for autologous breast reconstruction.

The thoracodorsal vessels, prior to development of the internal mammary vessels as the preferred option for autologous breast reconstruction, were commonly used for recipients, but utilizing these can negate the option of a latissimus dorsi as a salvage procedure in the setting of flap failure, especially if the plastic surgeon were to use the antegrade and retrograde vessels. Ipsilateral antegrade internal mammary and thoracodorsal vessels would be correct as well, but it was not given as an option. Thoracoacromial vessels are not typically used in autologous breast reconstruction, aside from being a lifeboat for venous outflow.

2021

79
Q

A 47-year-old woman comes to the emergency department because of wound separation. History includes immediate prepectoral breast reconstruction with placement of left tissue expander, and she is currently undergoing radiation therapy. Physical examination shows an approximately 0.5-cm area of incisional dehiscence with exposure of the device. Vital signs are stable, and inflammatory markers are mildly elevated. Which of the following is the most appropriate next step in management?

A) Admit the patient for intravenous antibiotic therapy and monitor the wound
B) Apply occlusive dressing and discharge the patient on oral antibiotic therapy
C ) Close the wound at bedside in the emergency department and discharge the patient on oral antibiotic therapy
D) Perform expander removal in the operating room
E) Perform operative replacement of the expander

A

The correct response is Option D.

Studies have shown that postmastectomy radiation therapy almost doubles the risk of complications, including wound breakdown. While attempts at implant salvage are tempting, they are often ultimately unsuccessful and may potentially delay necessary radiation treatments. Therefore, in the event of implant exposure during adjuvant therapy, the device should be removed and the mastectomy pocket closed over a surgical drain. This strategy will allow for the shortest delay in further oncologic treatments and does not preclude reconstruction at a later date. Closure of the wound over an exposed device should not be attempted, as any exposed device is assumed to be colonized, if not grossly infected. Although antibiotics may temporize the infectious process, it is unlikely that successful wound healing and retention of the prosthesis will occur after suppressive antimicrobials are stopped.

2021

80
Q

An otherwise healthy 41-year-old woman is scheduled to have bilateral prophylactic NSM for a BRCA1 mutation. Which of the following is an advantage of prepectoral implant reconstruction when compared with submuscular implant reconstruction?

A) There are significantly more acute postoperative surgical complications
B) There is a higher revision rate
C) There is a lower rate of animation deformity
D) There is a lower rate of upper pole rippling and implant palpability
E) There is more pain and functional impairment

A

The correct response is Option C.

The patient is a young, healthy, thin woman presenting for bilateral breast reconstruction following prophylactic mastectomies because of increased lifetime risk of breast cancer. She will maintain her entire breast skin envelope because she has planned nipple-sparing mastectomies and does not wish to increase her breast size with the operation. She has questions about the plane of implant placement above or below the pectoralis muscle and the answer choices provide the opportunity to counsel her on direct to implant versus staged expander to implant breast reconstruction.

Because of the lack of camouflage afforded by placement of the pectoralis muscle over the upper pole of the breast implant, prepectoral breast reconstruction is associated with increased rippling of the upper portion of the breast. This patient is thin according to BMI and so will have an increased risk of upper pole rippling that can be improved with subsequent fat grafting, provided she has adequate donor sites for fat harvest.

Several patient series reporting on early outcomes after prepectoral breast reconstruction have used a partially submuscular patient cohort as a comparison for acute postoperative complications. The overwhelming majority of these series showed no significant differences in acute postoperative outcomes and revision rates between techniques.

Postoperative functional assessments have demonstrated less pain and earlier return of function after prepectoral implant placement compared to submuscular implant placement. Re-siting of submuscular implants to a prepectoral plane have demonstrated resolution of animation deformity.

Algorithms for patient selection have slowly evolved over the past five years with the only constant recommendation being that the ideal candidate has a BMI less than 30 kg/m2. Earlier series emphasized mastectomy skin flap thickness, whereas more recent studies focus on skin flap viability. Earlier studies considered radiation treatment a contraindication, whereas more recent reports demonstrate the negative impact of radiation on pectoralis major fibrosis in submuscular reconstruction as compared with prepectoral implant placement.

2021

81
Q

A 43-year-old woman with a history of Stage I T1N0 invasive ductal carcinoma of the right breast presents 15 years after undergoing bilateral mastectomy, negative right sentinel lymph node biopsy, and staged expander to implant breast reconstruction. Over the past 3 months, her left reconstructed breast has become significantly larger than her right. On examination, there are no skin lesions, palpable masses, or axillary lymphadenopathy. Which of the following is the most appropriate next step in the workup of this patient?

A) Ultrasound
B) Mammography
C) PET-CT
D) MRI

A

The correct response is Option A.

The patient presents 15 years after bilateral mastectomy and staged expander to implant breast reconstruction with a new finding of left reconstructed breast enlargement. This may be due to a large late periprosthetic fluid collection, Baker Classification Grade III and IV capsular contracture can give the impression of breast enlargement because of the increased projection and vertical height of the breast contour. Any patient presenting with a late periprosthetic fluid collection should be evaluated for breast implant-associated anaplastic large cell lymphoma (BIA-ALCL). In this patient, it is unknown what type of implant she has; therefore, radiologic evaluation is warranted.

Physical examination and a breast ultrasound would distinguish between these two causes of perceived enlargement of a breast after implant-based breast reconstruction. Ultrasound can also identify capsular masses and provides an opportunity to aspirate the fluid collection, which often provides symptomatic relief for the patient, as well as an opportunity to perform cytology to analyze lymphocyte numbers and characteristics, in addition to flow cytometry to measure anaplastic lymphoma kinase negativity and CD30 cell surface receptor positivity.

Presentation with BIA-ALCL can take many years to develop, with a median 10 years to fluid collection development. Surgical treatment for BIA-ALCL can vary with the stage of presentation, but BIA-ALCL confined to the periprosthetic fluid can be effectively treated with capsulectomy and implant removal. The majority of cases of BIA-ALCL present with late-onset seroma (66-80%), whereas the second most common presentation is an isolated mass within the capsule around the implant.

PET-CT is often performed as part of a staging evaluation after the diagnosis of BIA-ALCL is confirmed. Mammography is a breast cancer screening tool, but has no role in post-mastectomy evaluation.

A prone breast MRI (or ultrasound) is recommended 5 to 6 years after silicone breast implant placement and every 2 years thereafter to evaluate the integrity of the implant shell, but has no role in the management of BIA-ALCL.

2021

82
Q

Lactiferous ducts in the human embryo are derived from which of the following types of cell populations?

A) Endodermal
B) Epithelial
C) Mesenchymal
D) Neural crest
E) Pluripotential

A

The correct response is Option B.

As early as 6 weeks of gestation, discrete areas of epithelial proliferation occur on the ventral thorax. These areas evolve into buds that then canalize and form secondary buds that give rise to lactiferous ducts. These, along with secretory acini that originate in puberty, constitute the parenchyma of the breasts. These ducts are surrounded by mesodermal-derived mesenchymal cells, which in turn develop into the breast stroma composed of smooth muscle cells, capillary endothelial cells, and adipocytes. Neural crest cells give rise to pigmented cells and other structures but are not involved with embryogenesis of the breasts. Endodermal cell lines produce the lining of the gut and contribute to development of the liver and pancreas but not the breasts. Pluripotential cells do not comprise the parenchyma nor the stroma of the developing breast.

2021

83
Q

Which of the following hormones in the mammary gland is critical for breast development during puberty?

A) Cortisol
B) Insulin-like growth factor-1 (IGF-1)
C) Platelet-derived growth factor (PDGF)
D) Prolactin
E) Vascular endothelial growth factor-C (VEGF-C)

A

The correct response is Option B.

While pubertal breast development depends on an estrogen surge, it is important to remember that breast development occurs downstream. The surge triggers pituitary growth hormone to stimulate mammary gland production of insulin-like growth factor-1 (IGF-1). IGF-1, in turn, causes proliferation and anti-apoptosis through a signaling cascade. Disruption of any factor along this pathway will result in abnormal breast development. Laron syndrome, for example, is an autosomal recessive disorder in which growth hormone insensitivity can cause delayed puberty as well as short stature. While high levels of prolactin can cause breast growth (and are thought to be responsible for ultimate breast development in Laron syndrome), prolactin is not in the mammary glands themselves.

In addition to the critical role IGF-1 plays during breast development, it also plays an important role in breast development during pregnancy. High levels of IGF-1 are also associated with increased breast volume and early-onset breast cancer. High serum levels of IGF-1 are associated with both bad prognosis and increased mortality in breast cancer. Cortisol is a stress hormone not related to breast development. Platelet-derived growth factor has a primary role in blood vessel development, as does vascular endothelial growth factor (VEGF). VEGF-C has a function in lymphangiogenesis.

2021

84
Q

A 54-year-old woman presents for follow-up examination after undergoing left mastopexy for improved symmetry 6 months ago. Medical history includes breast cancer. The patient reports some discomfort overlying a firm 1.5-cm mass that is palpable in the subcutaneous plane of the left breast. Which of the following is the most appropriate next step in management?

A) Fine-needle aspiration
B) Surgical excision
C) Ultrasound imaging
D) Warm compresses
E) Observation only

A

The correct response is Option C.

The most appropriate next step in management is to obtain imaging. Ultrasound can identify and characterize lesions such as fat necrosis, which is the most likely cause of this patient’s palpable mass in the postoperative period. Such imaging can confirm the physical examination findings and guide the best course of treatment (ie, aspiration versus excision). Observation or warm compresses are inadequate in addressing these findings or in obtaining a diagnosis. Once the benign nature and characteristics of the lesion are better understood, then treatment with bedside aspiration or formal excision of the mass can be undertaken.

2021

85
Q

According to the 2017 US breast cancer mortality data, three states with appropriately powered data achieved statistically equivalent mortality rates between non-Hispanic African American and non-Hispanic Caucasian women. Which of the following factors was most likely implicated in the improvement in mortality rates in the non-Hispanic African American women in these states?

A) Non-Hispanic African American women are more affluent
B) Non-Hispanic African American women are younger
C) Non-Hispanic African American women have fewer “triple-negative” aggressive breast cancers
D) Non-Hispanic African American women have more access to screening mammography and in situ diagnosis

A

The correct response is Option D.

Social determinants of health include where a person resides, what exposures are present, what diet is eaten, how much stress is present, and other lifetime experiences. Social determinants of health affect who gets disease and how well they can be cared for when they have disease.

According to the 2020 cancer statistics, death from female breast cancer has declined overall by 40% in the United States since 1989.

The 2017 breast cancer statistics look at breast cancer in a state-by-state fashion demonstrating that those gains in survival have not been uniformly spread among the populations, with non-Hispanic African American women suffering higher death rates by age group, staging, and oncotyping groups.

While there are no data to discuss why non-Hispanic African American women tend to develop cancer younger or develop more “triple negative” breast cancers, there has been evidence presented in the 2017 breast cancer statistics that better access to care may improve the stage at diagnosis. In seven states, the mortality rates were statistically the same. In three of those states—Massachusetts (an early state to establish mandatory health insurance), Connecticut, and Delaware—there were sufficient cohort numbers of non-Hispanic African American women with an in situ diagnosis, which is used as a proxy for access to screening mammography. In the four other states, there was statistical equivalence, but they were underpowered. The implication of these data is that better access to mammography/care lowers mortality for non-Hispanic African American women with breast cancer. These studies did not address affluence, but they did rule out age as a factor.

2021

86
Q

A 55-year-old woman has ductal carcinoma in situ located approximately 6 cm from the nipple on the lateral upper quadrant of the left breast. Which of the following is the minimum margin of excision if she decides to proceed with lumpectomy followed by radiation therapy?

A) 1 mm
B) 2 mm
C) 5 mm
D) 1 cm
E) 2 cm

A

The correct response is Option B.

Consensus guidelines recommend a 2-mm minimum margin for patients undergoing breast-conserving therapy. Margins over 2 mm are not associated with further risk of local recurrence.

Primary treatment options for women with ductal carcinoma in situ (DCIS) are lumpectomy plus whole breast radiation with or without boost, total mastectomy - with or without SLNB with optional reconstruction, or lumpectomy alone. For patients with DCIS treated with breast conserving therapy, which includes lumpectomy followed by radiation, margins of at least 2 mm are associated with a reduced risk of ipsilateral breast tumor recurrence relative to narrower negative margin widths. However, a negative margin width wider than 2 mm is not supported by the evidence. An analysis of specimen margins and specimen radiographs should be performed to ensure that all mammographically detectable DCIS has been excised. In addition, a postexcision mammogram should be considered where appropriate.

2021

87
Q

A 56-year-old woman with breast cancer undergoes bilateral mastectomy and immediate breast reconstruction with deep inferior epigastric perforator (DIEP) flaps. She has an uneventful recovery and is discharged home on postoperative day 4. Which of the following best describes the expected postoperative changes to the lower extremity venous system during this patient’s hospital stay?

A) There is decrease in diameter of the common femoral vein
B) There is increase in flow velocity of the common femoral vein
C) There is no change to the venous circulation
D) There is persistent venous stasis through the day of discharge
E) There is venous stasis, influenced by unilateral versus bilateral flap reconstruction

A

The correct response is Option D.

There is an increased risk of deep venous thrombosis associated with autologous breast reconstruction. Studies have tried to correlate autologous breast reconstruction with decreased venous return and stasis.

When compared with baseline levels, the common femoral veins have increased diameter and decreased flow velocity. These changes persist through the day of discharge. Since lower abdominal tissue is harvested and tight abdominal closure is performed in both unilateral and bilateral autologous breast reconstruction, there is no difference in venous stasis postoperatively when comparing both groups.

2021

88
Q

Which of the following legislative acts mandates insurance coverage of breast reconstruction after total mastectomy as well as coverage of any associated symmetry procedures for the contralateral breast?

A) Affordable Care Act
B) Breast Cancer Patient Education Act
C) SB-255 Amendment to the Knox-Keene Health Care Service Plan Act
D) Women’s Health and Cancer Rights Act

A

The correct response is Option D.

The Women’s Health and Cancer Rights Act of 1998 (WHCRA) is a federal law that mandates the coverage of breast reconstruction after mastectomy as well as coverage of any associated symmetry procedures for the contralateral breast. Although this federal law was enacted more than 20 years ago, there are still significant disparities in access to breast reconstruction and a lack of education regarding the options available for breast reconstruction.

The Breast Cancer Patient Education Act of 2015 is a federal law that requires the Secretary of Health and Human Services to implement an educational campaign to inform breast cancer patients about access, availability, and options for breast reconstruction after mastectomy. SB-255 is an amendment in the state of California to the Knox-Keene Health Care Service Plan Act that includes “lumpectomy” for treatment of breast cancer in the definition of “mastectomy” and mandates access to insurance coverage of breast reconstruction after lumpectomy in the state of California. It is important to note that the WHCRA only mandates coverage of breast reconstruction after mastectomy, not after breast conservation therapies such as lumpectomy. In fact, there is no current legislation mandating insurance coverage for all types of breast reconstruction, including breast reduction, mastopexy, or implant complications after aesthetic surgeries.

The Affordable Care Act (ACA) is a comprehensive health care reform law enacted in March of 2010. The three primary goals of the ACA are to make affordable health insurance available to more people, to expand the Medicaid program to cover all adults with income below 138% of the federal poverty level, and to support innovative medical care delivery methods designed to lower the costs of health care generally. There is nothing specifically in reference to breast cancer or breast reconstruction care within the ACA.

2021

89
Q

An otherwise-healthy 45-year-old woman presents for discussion regarding autologous breast reconstruction following mastectomy. Medical history includes abdominoplasty following the birth of her last child. After examination, the surgeon determines that she is a good candidate for bilateral lumbar artery perforator flaps. During the preoperative discussion, the surgeon compares the risks, benefits, and expected outcomes between lumbar artery perforator flaps and deep inferior epigastric artery perforator (DIEP) flaps, which are the surgeon’s typical initial choice for autologous tissue transfer. Which of the following complications is associated with lumbar artery perforator flaps when compared with DIEP flaps?

A) Higher breast dissatisfaction rate
B) Higher hematoma rate
C) Higher secondary revision rate
D) Higher total flap loss rate

A

The correct response is Option D.

Lumbar artery perforator (LAP) flaps have recently been shown to be a good autologous breast reconstruction option when deep inferior epigastric perforator (DIEP) flaps are not available. LAPs do carry a higher total flap loss rate with reports of 6.6 to 9% compared with less than 2% for DIEPs. Based on the microvascular compromises, which are reported to be more venous than arterial, the take-back rate on LAPs is also expectantly higher than DIEPs. Hematoma and secondary revision rates are very similar between the two groups. Based on BREAST-Q data, the satisfaction rate in the breast reconstruction is also very similar between the two groups. This is not the case in satisfaction rate of the donor site, where responses were lower in LAPs compared with DIEPs. This is likely due to a higher donor site complication rate in the form of seromas and postoperative pain.

2022

90
Q

A 42-year-old woman presents 1 week following exchange of tissue expander for permanent implant, in which skin glue was used for closure. She reports an itchy rash around her incision. Medical history includes right breast cancer, for which she has undergone mastectomy and placement of a pre-pectoral textured tissue expander with acellular dermal allograft coverage. She is afebrile. A photograph is shown. Which of the following is the most likely cause of this rash?

A) Anaplastic large-cell lymphoma infiltration of the skin
B) Antiphospholipid antibody-mediated urticaria
C) Cyanoacrylate exposure and dermatitis
D) Lymphatic disruption leading to red breast syndrome
E) Staphylococcus aureus cellulitis

A

The correct response is Option C.

The patient has recently had surgery and is doing well except for her skin rash. The rash appears to be localized to the incision. Contact dermatitis to surgical adhesives is a common problem. There is growing recognition that surgical glues containing cyanoacrylate may be associated with a 10 to 15% rate of rashes secondary to Type IV allergic hypersensitivity reactions. These typically appear within 3 to 7 days and are treated by removal of the surgical glue and topical steroid cream. Patients who have previously been exposed to surgical glue are more at risk for developing sensitivity.

Surgical site infection is a concern following implant placement. However, a Staph cellulitis is typically more diffuse and has a different appearance more consistent with erythema rather than a rash. Antiphospholipid syndrome (APS) may be associated with a rash and other systemic symptoms; however, there is no indication within the history that this patient suffers from that condition. Additionally, the timing and location of the rash along the incision point to other local causes rather than APS. Lymphatic disruption is one theory to explain red breast syndrome, which is a condition that has been reported in relation to acellular dermal matrix placement. This is typically more diffuse and usually occurs in the acute setting following acellular dermal matrix placement. Anaplastic large-cell lymphoma is a rare type of cancer that may be associated with breast implants and may present with swelling and a rash. There is a higher rate of this condition in patients who have had textured devices. The mean onset for this condition is 8 years after exposure to the breast implant. This patient’s clinical course is not typical of anaplastic large cell lymphoma.

2022

91
Q

A 56-year-old woman presents with significant capsular contracture and deformity of her left reconstructed breast 3 years after undergoing bilateral mastectomy, left-sided radiation therapy, and subsequent two-stage implant reconstruction. A photograph is shown. She has a history of abdominoplasty. Which of the following is the most reliable way to achieve an acceptable reconstruction in this patient?

A) Conversion to prepectoral plane with acellular dermal allograft
B) Deep inferior epigastric perforator flap
C) Implant exchange with capsulectomy and acellular dermal allograft
D) Implant exchange with capsulotomy
E) Latissimus dorsi flap with expander

A

The correct response is Option E.

In this patient with a history of left-sided radiation and a severely contracted and deformed implant, the most reliable way to achieve an acceptable reconstruction would be to convert to some form of autologous reconstruction. Given her prior history of abdominoplasty, a deep inferior epigastric perforator flap is not feasible. A latissimus dorsi flap with an expander is a reliable option. Implant exchange with capsulotomy would have an extremely high risk for recurrence of capsular contracture and deformity. Capsulectomy with acellular dermal allograft replacement would have a fairly high risk for complications due to adding yet more foreign body to a radiated tissue bed, and there would still be a high risk for recurrent capsular contracture. Conversion to a prepectoral plane with acellular dermal allograft would have a potentially higher risk for complications because even more foreign body is required to fully wrap the implant.

2022

92
Q

A 76-year-old woman presents with swelling of her right reconstructed breast, which was performed with a textured implant 14 years ago. Ultrasound demonstrates an uncomplicated fluid collection. The patient denies pain, fevers, or chills. Enrichment of which of the following cell surface markers within the aspirated fluid is suggestive of breast implant-associated anaplastic large cell lymphoma?

A) CD3
B) CD15
C) CD30
D) CD34
E) CD45

A

The correct response is Option C.

Breast implant-associated anaplastic large cell lymphoma (BIA-ALCL), a type of T-cell lymphoma, is associated with textured implants. Initial diagnosis is often made following ultrasound. CD30 is a cell surface marker detected on T-cells associated with BIA-ALCL. The other cell surface markers are not associated with BIA-ALCL. These cells are also anaplastic lymphoma kinase (ALK) negative. CD3 and CD15 are nonspecific lymphocyte markers not related to BIA-ALCL. CD34 is an endothelial cell marker. CD45 is a marker expressed on all leukocytes.

2022

93
Q

A 45-year-old woman presents for bilateral immediate breast reconstruction. An abdominal flap is planned. The patient demands reconstruction with a flap that has the lowest abdominal wall morbidity since she is a yoga and pilates instructor. Which of the following autologous flaps utilized for breast reconstruction results in the lowest rate of abdominal wall morbidity?

A) Deep inferior epigastric perforator flap
B) Free transverse rectus abdominus myocutaneous flap
C) Muscle-sparing free transverse rectus abdominus myocutaneous flap
D) Pedicled transverse rectus abdominus myocutaneous flap
E) Superficial inferior epigastric artery flap

A

The correct response is Option E.

Of all the abdominally based flaps listed for breast reconstruction, only the superficial inferior epigastric artery (SIEA) flap does not violate the abdominal fascia. Each of the other flaps are associated with abdominal bulge or hernia formation. Pedicled and free transverse rectus abdominus myocutaneous (TRAM) flaps both involve the harvest of significant abdominal muscle and are associated with more abdominal wall dysfunction than with deep inferior epigastric perforator (DIEP) flaps. It is unclear whether there is a difference in abdominal wall morbidity following muscle-sparing free TRAMs versus DIEPs. However, only the SIEA flap offers no muscle dissection or violation of the abdominal fascia.

2022

94
Q

A 40-year-old woman with cancer of the right breast undergoes a bilateral skin-sparing mastectomy and immediate reconstruction with placement of submuscular tissue expanders. After completion of the bilateral mastectomies, the breast skin flaps appear to be dusky with some bruising of the skin edges. Which of the following pharmacologic agents has been shown to decrease the rate of mastectomy flap necrosis?

A) Indocyanine green
B) Lidocaine with epinephrine
C) Nitroglycerin paste
D) Papaverine

A

The correct response is Option C.

Nitroglycerin paste has been shown to decrease the rate of mastectomy flap necrosis.

Topical nitroglycerin ointment application has been shown in a prospective randomized clinical trial to decrease the incidence of mastectomy flap necrosis in immediate tissue expander breast reconstruction.

Papaverine is incorrect since it is a vasodilator that relaxes smooth muscles in blood vessels. This agent is useful during microvascular surgery to decrease vasospasm of arteries and veins but has not been used to decrease the rates of mastectomy flap necrosis.

Indocyanine green is incorrect since it is a dye used for diagnostic purposes during fluorescence angiography. It can be administered intravenously and be used to assess tissue perfusion intraoperatively, which can be helpful for diagnosis of mastectomy flap necrosis. It does not change tissue perfusion, however.

Lidocaine with epinephrine is incorrect since it is an analgesic agent with a vasoconstrictor. This can be used intraoperatively for pain control and hemostasis. At more dilute concentrations, it can be used for tumescent injection during mastectomy. This has not been shown to decrease the rates of mastectomy flap necrosis.

2022

95
Q

A 62-year-old woman undergoes left mastectomy with axillary lymph node dissection and tissue expander reconstruction. A drain is placed adjacent to the tissue expander and in the axilla. An intravenous catheter is placed in her left arm, and blood pressure is measured from same arm. BMI is 42 kg/m2. Which of the following risk factors is most significantly associated with lymphedema in this patient?

A) Compression from a blood pressure cuff
B) Drain placement in the axilla
C) Obesity
D) Placement of an intravenous catheter
E) Tissue expander reconstruction

A

The correct response is Option C.

Obesity (BMI >30 kg/m2) is the most significant risk factor for lymphedema from the listed choices. Patients who undergo axillary lymphadenectomy and radiation are at an approximately 30% risk for acquiring lymphedema. Obese individuals are at higher risk. In addition, obesity-induced lymphedema is a phenomenon in which patients who reach a BMI of 50 to 60 kg/m2 have a high risk for lymphedema, in particular bilateral lower extremity lymphedema, without any other inciting events.

Compression from a blood pressure cuff is unlikely to cause lymphedema. Lymphedema is primarily treated with compression (pneumatic, static). Evidence is lacking that an intravenous catheter increases the risk for lymphedema. Tissue expander reconstruction or any breast reconstruction does not increase the risk for lymphedema and neither does drainage tube placement.

2022

96
Q

An otherwise healthy 45-year-old woman undergoes bilateral breast reconstruction with free deep inferior epigastric perforator-based flaps. Only the medial deep inferior epigastric vein was anastomosed to the medial internal mammary vein using a 3.5-mm venous coupler. Her right breast flap has become increasingly congested. The cutaneous Doppler signals are strong, and examination of the pedicle shows good flow through both arterial and venous anastomoses. There is no kinking or hematoma. Which of the following is the most appropriate next step in management?

A) Additional anastomosis of the lateral deep epigastric vein
B) Additional anastomosis of the superficial inferior epigastric vein
C) Revise the arterial anastomosis
D) Revise both anastomoses
E) Revise the venous anastomosis

A

The correct response is Option B.

The most appropriate next step in management is to supercharge the flap using an additional anastomosis of the superficial inferior epigastric vein. This is a clinical example of persistent superficial venous system dominance. This is due to either the superficial system being the dominant venous drainage of the abdominal wall and the absence of connections between the superficial and deep venous systems, or the lack of an adequate number of perforators in the flap. In either case, the venous blood is not getting from the superficial system into the deep system, which is the egress given that the deep system is the only system anastomosed.

There is no technical issue with the anastomosis or venous thrombosis; therefore, revising the deep anastomoses is not required. Rather, it is an intra-flap venous system issue that requires the superficial system to be drained somehow. This is accomplished by anastomosing the superficial inferior epigastric vein to either the deep system in the flap or to another drainage system in the chest like the intercostal system. It is interesting to note that the two reasons for the above issue (i.e., inadequate perforator selection or anatomic lack of communicating vessels between the superficial and deep system), though distinct entities, clinically present similarly and are treated in the same way. The incidence of venous congestion secondary to persistent superficial system dominance is about 0.9%.

2022

97
Q

A 45-year-old woman undergoes bilateral nipple-sparing mastectomy with immediate tissue expander reconstruction for T3N2 breast cancer. She completes radiotherapy and resumes expansion in the office 4 months after surgery. Her husband calls the office noting that the patient is reporting headache and requests additional pain medication when she suddenly demonstrates seizure activity. She is taken to the emergency department for immediate evaluation and stabilization. Which of the following imaging modalities should be used with caution for this patient?

A) CT scan
B) Fluoroscopy
C) MRI
D) Positron emission study
E) Ultrasound

A

The correct response is Option C.

The majority of breast tissue expanders include a ferromagnetic port and are currently labeled as MRI-unsafe due to the potential interaction with the magnetic field of the machine. There are reports of several MRI-related complications: malposition, pain or burning sensation, polarity reversal, port dislodgement, and thermal injuries. However, several subsequent studies have demonstrated modifications to minimize risks in patients requiring diagnostic MRI of either the contralateral breast, perforator mapping, or evaluation of brain/spine metastases. This patient may need MRI evaluation of the brain to diagnose metastatic disease of the brain. While some imaging technicians may be unwilling to perform MRI due to the warnings, studies suggest use of a 1.5T MRI, saline filling, and prone positioning are effective strategies to minimize risks and safely perform MRI rather than explant the device(s).

CT scanning can be safely performed with no contraindication due to expanders. Positron emission studies also do not interfere with expanders. Ultrasound is frequently used to safely assess and treat possible fluid collections or infections associated with expanders. Despite her previous radiation therapy, fluoroscopy can still be safely performed while taking care to minimize radiation exposure to critical structures.

2022

98
Q

An otherwise healthy 45-year-old woman develops an infection following implant-based breast reconstruction. Which of the following gram-negative species is most likely to be involved with the infection?

A) Enterobacter spp
B) Escherichia coli
C) Proteus spp
D) Pseudomonas spp
E) Staphylococcus spp

A

The correct response is Option D.

The most likely gram-negative species associated with implant-based breast reconstruction infection is Pseudomonas spp.

In general, gram-positive organisms (about 70%) are most commonly associated with such infections, in which Staphylococcus species are the most commonly cultured (51%). Gram-negative species are associated with about 27% of the infections, of which Pseudomonas were the most commonly cultured. These data guide the empiric antibiotics that should be used initially at presentation. Staphylococcus species are the most common bacteria involved with breast implant infections; however, they are gram-positive bacteria.

2022

99
Q

A 56-year-old woman presents with right breast swelling 10 years after undergoing bilateral mastectomy and reconstruction with bilateral submuscular textured implants with an acellular dermal matrix sling. Examination shows right-sided periprosthetic fluid collection. Breast implant-associated anaplastic large cell lymphoma is suspected. Analysis of the fluid will most likely show which of the following tumor markers?

A) Anaplastic lymphoma kinase
B) CD3
C) CD5
D) CD30
E) Human germinal center-associated lymphoma (HGAL)

A

The correct response is Option D.

Breast implant-associated anaplastic large cell lymphoma (BIA-ALCL) has been associated primarily with textured implants. Thought to be related to chronic inflammation from subclinical infection, the presentation is typically with a late-onset seroma. Workup of a late-onset seroma includes aspiration of the seroma and cytological/immunohistochemical analysis. The predictive value is most significant on the initial aspiration because of the presumed larger concentration of malignant cells; as the seroma reforms, there is a hypothesized dilutional aspect.

BIA-ALCL neoplastic cells are strongly CD30-positive, but CD30 is a necessary but insufficient criterion to make the diagnosis since it can be displayed in nonmalignant situations as well (such as in viral infection).

While anaplastic lymphoma kinase (ALK) is associated with systemic ALCL in 60% of cases, it has not been associated with BIA-ALCL. Thus, while ALK assessment is considered necessary to the analysis, it is expected to be negative. BIA-ALCL neoplastic cells variably lose expression of CD3 and CD5. Compared with other forms of ALCL, BIA-ALCL is associated with a more indolent course, although a subset of patients do have an aggressive course. Human germinal center-associated lymphoma (HGAL) is associated with B-cell lymphoma.

2022

100
Q

A 49-year-old woman who underwent first-stage left breast reconstruction with a tissue expander presents for follow-up to discuss expander to implant exchange. The reconstructed side has more volume and superior fullness compared with the native breast, and the patient prefers the reconstructed side. A photograph is shown. In addition to implant exchange, which of the following would most likely give the patient the greatest satisfaction regarding the appearance of her breasts?

A) Left breast fat grafting
B) Left breast flap reconstruction
C) Right breast implant augmentation
D) Right breast mastopexy
E) No additional procedures

A

The correct response is Option C.

The patient shown in the initial photograph complains of lack of symmetry between the reconstructed breast and the native, contralateral side. The volume and superior fullness in the reconstructed breast are a result of an implant-based reconstruction. Studies have shown improved patient satisfaction with breast reconstruction when contralateral augmentation is performed for symmetry. For this patient who was happy with an implant-based reconstruction, symmetry was best achieved with a contralateral breast augmentation. In the additional image shown, she is pictured after tissue expander to implant exchange, nipple-areola complex reconstruction, and right breast augmentation. The other choices are less ideal and would not necessarily give the patient the symmetry or satisfaction she desires.

2022

101
Q

Breast cancer cells can spread directly to the cranial cavity and brain via the vertebral venous plexus (Batson plexus). Through which of the following veins is it possible for these cells to reach this plexus?

A) Axillary
B) Azygos
C) Brachiocephalic
D) Intercostal
E) Internal thoracic

A

The correct response is Option D.

Hematogenous spread through the intercostal veins is the most likely way for breast cancer to reach the internal vertebral venous plexus. Although 75% of the lymph from the breast drains to the axillary nodes with most of the remaining lymph draining to the parasternal nodes, these lymphatic channels are not the most likely way to transmit cancer to the internal vertebral venous plexus. This plexus of veins would be most likely to receive cancer cells transmitted through the blood. The internal thoracic vein drains blood from the breast in the opposite direction of the vertebral column. The thoracoacromial artery supplies blood to the breast, as opposed to draining the breast, and therefore would not provide a route for spreading cancer.

2022

102
Q

A 51-year-old woman presents for breast reconstruction. She was recently diagnosed with left-sided breast implant-associated anaplastic large cell lymphoma (BIA-ALCL). She has bilateral submuscular textured implants that were placed 8 years ago. The patient would also like to have the contralateral implant removed during the left-sided surgery. Which of the following best describes the chance of finding BIA-ALCL pathology in this patient’s contralateral breast?

A) 1.6%
B) 4.6%
C) 7.6%
D) 10.6%
E) 13.6%

A

The correct response is Option B.

The risk for breast implant-associated anaplastic large cell lymphoma (BIA-ALCL) has been estimated to be between 1:1000 and 1:30,000 women with textured breast implants. Management consists of removal of the implant, complete surgical excision of the surrounding implant capsule, and excision of any suspicious lymph nodes. According to National Comprehensive Cancer Network guidelines, removal of the contralateral implant can be considered because in 4.6% of reported cases, the pathology disclosed BIA-ALCL in the contralateral breast.

Once full diagnostic workup and staging are performed, the options for reconstruction need to be discussed with the patient. A patient with resectable disease, stage IA to stage IC, can be offered resection and immediate reconstruction with either a smooth implant or autologous reconstruction, or the patient can choose to have resection followed by repeat imaging in 3 to 6 months and delayed reconstruction.

Stage IA is T1 N0 M0 disease confined to effusion or a layer on the luminal side of the capsule with no lymph node involvement and no distant spread.

Stage IC is T3 N0 M0 cell aggregates or sheets infiltrating the capsule, no lymph node involvement, and no distant spread.

In advanced disease, stages IIA to IV, surgical resection and chemotherapy followed by repeat positron emission tomography CT scan in 6 to 12 months is recommended. If the patient has no signs of metastatic activity, then delayed reconstruction with either a smooth implant or autologous reconstruction are both options.

2022

103
Q

A 45-year-old woman with a history of breast cancer presents for a discussion of breast reconstruction with silicone breast implants. She inquires about the current screening recommendations for implant rupture. Which of the following postoperative screening recommendations should the surgeon communicate to this patient?

A) CT scan at 3 years followed by CT, MRI, or ultrasound every 2 years thereafter
B) Mammogram every 5 years
C) MRI at 2 years then every 3 years thereafter
D) MRI at 3 years then every 2 years thereafter
E) MRI or ultrasound at 5 to 6 years then every 2 to 3 years thereafter

A

The correct response is Option E.

As a critically important part of the informed consent process, practitioners should be very familiar with updated information from the FDA. The screening recommendations for implant rupture for women with silicone gel-filled implants were recently updated (September 2020). The current FDA recommendations are that asymptomatic patients should have the implants evaluated with either ultrasound or MRI 5 to 6 years after implantation and then every 2 to 3 years thereafter. This new recommendation replaces the old recommendation by adding ultrasound as an accepted modality and extending the time until the first evaluation. The older recommendation was MRI evaluation at 3 years, with follow-up evaluation being performed every 2 years. These recommendations apply to women who have cosmetic augmentation and breast reconstruction. CT scan is not an acceptable modality for implant evaluation. Since this patient has had mastectomies and breast reconstruction, mammography is not indicated.

2022

104
Q

A 51-year-old woman with history of right breast invasive ductal carcinoma (IDC) status post right skin-sparing mastectomy, axillary node dissection, and adjuvant radiation therapy presents to discuss delayed breast reconstruction. Her recent positron emission tomography scan was negative for distant metastasis. Her pathology report shows positive estrogen receptor, positive progesterone receptor, negative human epidermal growth factor receptor, and an IDC 2.5 cm in size with three positive axillary lymph nodes. Based on these findings, which of the following is the TNM classification for this tumor?

A) T1 N2 M0
B) T1 N3 M1
C) T2 N1 M0
D) T2 N3 M0
E) T3 N2 M0

A

The correct response is Option C.

The tumor node metastasis (TNM) classification helps to stage tumors. Based on the information given about this breast cancer patient, she would be staged as T2 (primary tumor is 2.5 cm), N1 (three axillary nodes involved), and M0 (no evidence of metastatic disease on systemic radiologic workup. Please see the TNM classification system below.

Tumor staging: Tx - tumor cannot be assessed; T0 - no evidence of primary tumor; T1 - tumor less than 2 cm; T2 - tumor greater than 2 cm but less than 5 cm; T3 - tumor greater than 5 cm; T4 - tumor with direct extension into chest wall or skin.

Node staging: Nx - nodes cannot be assessed; N0 - no nodal involvement; N1 - one to three ipsilateral nodes involved; N2 - four to nine ipsilateral or internal mammary nodes involved; N3 - ten or more ipsilateral nodes.

Metastasis staging: Mx - distant spread cannot be assessed; M0 - no distant spread; M1 - cancer has spread to distant organs or liver.

2022

105
Q

An otherwise healthy 50-year-old woman with right-sided breast cancer undergoes autologous breast reconstruction. Which of the following free flaps has the highest risk for loss secondary to vascular thrombosis?

A) Deep inferior epigastric perforator
B) Gluteal artery perforator
C) Muscle-sparing transverse rectus abdominis myocutaneous
D) Superficial inferior epigastric artery
E) Transverse rectus abdominis musculocutaneous

A

The correct response is Option D.

The free flap that has the highest risk for vascular thrombosis is the superficial inferior epigastric artery (SIEA) flap.

There are numerous autologous free flap options for breast reconstruction. The most commonly performed is the deep inferior epigastric perforator flap. Others include variations of the transverse rectus abdominis musculocutaneous (TRAM) flap (standard TRAM or muscle-sparing TRAM), the SIEA flap, the gluteal artery perforator flap, lumbar artery perforator flap, and a number of thigh-based free flaps, like the gracilis musculocutaneous flap variations (transverse upper gracilis, vertical upper gracilis, diagonal upper gracilis), and the profunda artery perforator flap.

The SIEA flap has the advantage of being the only abdominally based flap that does not violate the rectus sheath, and thus has the lowest abdominal wall morbidity in terms of weakness, bulges, or hernias. However, due to its smaller vessel caliber, shorter pedicle length, and higher propensity for spasm, as well as the location of the vessel in the subcutaneous plane and at the edge of the flap, it has been shown to have a higher rate of vascular thrombosis and overall flap loss. Studies suggest the rate is as high as 17% and 7%, respectively, compared with less than 1% for the other options.

2023

106
Q

A 47-year-old woman with a history of abdominoplasty presents for discussion of delayed unilateral right breast reconstruction following mastectomy. The patient is interested in autologous options. After discussion with the patient, the plastic surgeon elects to use a horizontally oriented profunda artery perforator flap from the upper posterior thigh. In this area, the dominant perforator most commonly courses through which of the following muscles?

A) Adductor brevis
B) Adductor longus
C) Adductor magnus
D) Gracilis
E) Rectus femoris

A

The correct response is Option C.

The profunda femoris artery gives off multiple musculocutaneous and septocutaneous perforators to supply the thigh. In the area of a horizontally oriented profunda artery perforator flap, the dominant perforator travels through the adductor magnus muscle and enters the subcutaneous region of the flap skin and subcutaneous tissue posterior to the gracilis muscle. The perforator does not usually course through the other muscles listed.

2023

107
Q

A 55-year-old woman is evaluated because of an abnormal mammogram of the right breast. History includes neither an inherited gene mutation nor a strong family history of breast disease. She undergoes ultrasound-guided core needle biopsy, which discloses pleomorphic lobular carcinoma in situ. Which of the following is the most appropriate next step?

A) Chemotherapy
B ) Radiation
C) Repeat mammography in 6 months
D) Surgery
E) Observation only

A

The correct response is Option D.

Pleomorphic lobular carcinoma in situ (PLCIS) of the breast is an infrequent, more aggressive histopathologic subtype of classic lobular carcinoma in situ (LCIS). The frequency of upgrade to invasive lobular carcinoma, invasive ductal carcinoma, and ductal carcinoma in situ when PLCIS is found with core needle biopsy (CNB) ranges from 18 to 75%. Therefore, lumpectomy after CNB diagnosis of PLCIS is the standard of care to assess for potential upgrades. Typical LCIS can be managed with observation or chemoprevention. Repeated imaging with ultrasonography or mammography has no utility. Radiation is not indicated for local disease.

2023

108
Q

A 13-year-old girl is brought to the office by her mother because of a birthmark that has recently begun to grow since menarche. Physical examination shows the mass is well-circumscribed and nontender, and it lies within the inframammary fold. There is no evidence of an infectious process, and results of laboratory workup are normal for the patient’s age. Which of the following is the most likely diagnosis?

A) Mondor disease
B) Poland syndrome
C) Polythelia
D) Tubular breast deformity
E) Virginal breast hypertrophy

A

The correct response is Option C.

Breast tissue begins to develop from the ectoderm during the fifth week of gestation. Paired mammary ridges, consisting of thickened ectoderm, expand bilaterally from the axillae to the groin and undergo apoptotic regression during the seventh and eighth weeks of gestation. This leaves behind only primordial breast tissue on the ventral thorax, which penetrates the underlying mesenchyme during weeks 10 to 14. This mesenchymal connective tissue that now surrounds the breast ultimately forms the superficial fascial system and suspensory ligaments of Cooper. Canalization occurs at 20 weeks. Late in fetal development, the mammary pit forms from an invagination of the epidermis at the origin of the mammary gland, contributing to development of the nipple. Differentiation of the breast tissue continues until birth and resumes at puberty under hormonal influences. Once puberty begins with proliferation of glandular and adipose tissue, the breast becomes a pendant structure with its inferior border defined by the inframammary fold.

Remnants of the mammary ridge form a track from the axilla to the groin along which supernumerary nipples may form after failure of regression at 3 months’ gestation. This abnormality is not uncommon and has an incidence of 0.2 to 2.5%. These nipples are most commonly found below the breast but above the lower abdominal skin. They have been reported to be bilateral in 50% of patients and found in unusual locations such as the scapula, posterior thigh, head, and neck. Although this is usually no more than an aesthetic concern to the patient, the nipples may respond in a normal physiologic manner and be susceptible to thesame disease processes. Although there are reports of associated congenital abnormalities, no syndrome has ever been described. The most appropriate treatment for the majority of patients is simple excision.

The tubular breast may have a constricted base but does not have herniation of breast tissue through the areola, causing its enlargement. This rare disorder occurs sporadically, without a well-known estimate of its incidence or cause. The central abnormality is a ring of fascia beneath the areola constricting its radial outgrowth, causing herniation of tissue just posterior to the nipple-areola complex at the natural hiatus in the outer layer of the superficial fascia. This has been reported as bilateral in approximately 89% of patients, and asymmetry of more than 100 g is present in over two-thirds of these patients. Although the presentation may be aesthetic in nature, the psychological impacts of this disorder can be severe.

Poland syndrome is a combination of chest wall deformity and absent or hypoplastic pectoralis muscle and breast, associated with shortening and brachysyndactyly of the upper limb. Clinical presentation varies widely; therefore, reconstructive procedures have to be adapted to the deformity, the range including chest wall stabilization or augmentation, dynamic muscle transfer, nipple and areola repositioning, and augmentation mammaplasty using prosthesis or autologous tissue transfer.

Juvenile virginal hypertrophy presents in late childhood and early adolescence. Mammary hypertrophy appears to be related to end-organ responsiveness to estrogens and not increased circulating blood levels. It is the fatty and fibrous elements of the breast that increase, whereas the glandular component remains unchanged.

Mondor disease is a condition characterized by thrombophlebitis of the superficial veins of the breast and anterior chest wall. Initially this condition presents with rapid development of a subcutaneous red cord-like lesion that causes pain at an early stage and subsequently becomes a painless fibrous band.

2023

109
Q

A 35-year-old woman with the BRCA1 mutation wants bilateral prophylactic mastectomies and is judged to be a good candidate for nipple-sparing mastectomy. Which of the following incisions for this procedure is associated with the highest risk for nipple-areola complex ischemia?

A) Horizontal radial incision
B) Inferior mammary fold incision
C) Inferior radial incision
D) Periareolar

A

The correct response is Option D.

Of all the incisions used for nipple-sparing mastectomy, the periareolar incision, though preferred by breast surgeons, removes 50% of the blood supply to the nipple and is associated with a higher risk for nipple-areola complex ischemia.

After nipple-sparing mastectomy, the blood supply to the nipple-areola complex is dependent on the subdermal plexus that forms an anastomotic network around the areola from the medial and lateral sides. A periareolar incision disrupts this network, decreasing its blood supply. Similarly, horizontal and inferior radial incisions extending to the nipple-areola complex disrupt the subdermal plexus, increasing the risk for nipple-areola complex ischemia.

The inferior mammary fold (IMF) incision, especially a lateral IMF incision, preserves some of the inframammary perforators medially and is associated with a lower incidence of nipple areola complex ischemia

2023

110
Q

A 25-year-old woman requests removal of multiple small bumps near the margin of the areolae bilaterally because she believes they signify breast cancer. On examination, the plastic surgeon determines that the concerning areas are normal physiological structures of the areolae. The most likely outcomes if these structures are removed is a decrease in which of the following aspects of the areola?

A) Contractility
B) Pigmentation
C) Sensation
D) Size
E) Skin lubrication

A

The correct response is Option E.

The most likely outcome if these normal structures are removed is decreased areola skin lubrication. The nipple-areola complex contains specialized structures including the termination of the lactiferous ducts and the Montgomery glands or tubercles. Montgomery glands or tubercles are accessory glands that vary in number and size and are often noted near the margin of the areola. Should these be removed, there is a potential for decreased quality of the skin of the areola. The character of these structures is not related to breast cancer. Limited removal of the structures without disruption of other anatomical components of the nipple areola complex would not decrease nipple reactivity, areola sensation, or contractility of the areola as these are controlled by nerve and smooth muscle mechanisms rather than glands.

2023

111
Q

A 15-year-old girl is brought to the plastic surgeon because of breast asymmetry. Examination shows a minimally tender, mobile mass of the upper outer quadrant of the left breast that measures approximately 4 cm in size. The patient reports that the mass has been present for 2 years. Which of the following is the most appropriate next step in management?

A) Excisional biopsy
B) Incisional biopsy
C) Mammography
D) MRI
E) Ultrasonography

A

The correct response is Option E.

The mass is most likely a fibroadenoma. Fibroadenomas are commonly located in the upper outer quadrant of the breast in adolescents and are circumscribed and mobile on physical examination. They are the most common breast mass in adolescents and commonly regress. If they are less than 5 cm in size, they can be observed. Giant fibroadenomas are characterized by sizes greater than 5 cm and should be excised.

Fibroadenomas that persist should be imaged, and the preferred imaging modality for adolescents is ultrasonography. Mammography confers a radiation dose that is not necessary for diagnosis, and adolescent breast tissue is dense. MRI is not necessary for diagnosis. While biopsy may ultimately be necessary, it is not performed prior to imaging.

2023

112
Q

A 16-year-old girl is referred for evaluation of breast asymmetry. She wears a size 36C brassiere, but she notes that the right breast does not fill the cup. On examination, the left breast appears normal, and the right breast has no identifiable nipple-areola complex or palpable glandular tissue. The pectoralis muscle and ribs appear normal. The patient’s development is consistent with Tanner stage 4. Which of the following is the most likely diagnosis?

A) Amastia
B) Amazia
C) Anterior thoracic hypoplasia
D) Athelia
E) Poland syndrome

A

The correct response is Option A.

Amastia is the complete absence of the nipple-areola complex and glandular breast tissue. Athelia is the absence of the nipple-areola complex; however, the glandular tissue is present. Amazia is absence of the breast tissue, but a normal nipple-areola complex is present.

In Poland syndrome, one finds varying degrees of thoracic and ipsilateral upper extremity anomalies. This typically includes some degree of breast hypoplasia or absence, and partial or complete absence of the pectoralis muscle. More involved cases may have the absence of ribs and the presence of digital abnormalities (eg, syndactyly, brachydactyly).

Anterior thoracic hypoplasia consists of posterior displacement of the ribs, anterior unilateral sunken chest wall, hypoplasia of the breast on the involved side, and superior displacement of the nipple-areola complex. The sternum and pectoralis major are normal.

2023

113
Q

A 70-year-old woman is evaluated because of left-sided breast cancer diagnosed after workup for persistent erythema of the skin. History includes type 1 diabetes (hemoglobin A1C concentration of 5%). The patient wears a D-cup brassiere, and BMI is 34 kg/m2 . Examination of a specimen obtained on biopsy demonstrates tumor cells infiltrating the dermis. Which of the following risk factors is the strongest indicator for delayed, rather than immediate, breast reconstruction in this patient?

A) Advanced age
B) BMI category
C) Breast size
D) Cancer type
E) Medical comorbidity

A

The correct response is Option D.

The risk factor most strongly indicating delayed, rather than immediate, breast reconstruction is cancer type. The described findings of erythema of the skin and biopsy demonstrating tumor cells infiltrating the dermis are consistent with inflammatory breast cancer. Guidelines recommend against immediate breast reconstruction at the time of surgical treatment for inflammatory breast cancer because of the high likelihood of recurrence and severity of disease, which would make potential delay in adjuvant therapy more concerning. While age, higher BMI, breast size, and medical comorbidities may be considered potential contraindications, the severity of these factors in the scenario are not usually considered strict contraindications to immediate breast reconstruction.

2023

114
Q

A 60-year-old woman who is considering silicone implant-based postmastectomy reconstruction asks about what she should expect for postoperative surveillance imaging. Which of the following is the best recommendation for asymptomatic surveillance in this patient?

A) High-resolution ultrasonography 5 years postimplantation
B) Implant displacement mammography 1 year postimplantation
C) Monthly self-examinations of the breasts
D) Noncontrast CT scan 7 years postimplantation
E) Noncontrast MRI 3 years postimplantation

A

The correct response is Option A.

The most accurate recommendation for asymptomatic silicone implant surveillance is highresolution ultrasonography 5 years postimplantation. The FDA recently issued updated asymptomatic silicone implant surveillance imaging recommendations. These state that asymptomatic patients may undergo their first imaging 5 to 6 years after silicone implantation using ultrasonography or MRI, followed by imaging every 2 to 3 years afterward. The FDA recommends that MRI is preferentially used for symptomatic patients or those with equivocal ultrasonography results. Monthly self-examinations of the breasts and mammography are more appropriate screenings for breast cancer. The recommendation for an MRI at 3 years postimplantation is now outdated. No recommendations exist regarding CT scans, and the 7 year postimplantation time frame is longer than recommended for initial imaging.

2023

115
Q

A 25-year-old woman presents to a plastic surgeon to discuss breast reconstruction. She is a carrier of the BRCA1 mutation and has decided to undergo risk-reducing mastectomies. Which of the following is the approximate average risk for this patient developing breast cancer by the age of 70 years if she does not undergo bilateral mastectomies?

A) 20%
B) 40%
C) 60%
D) 90%

A

The correct response is Option C.

In this clinical scenario of accurate counseling, it is important to know the risk for developing cancer in women with the BRCA1 and BRCA2 mutations. Based on multiple epidemiological studies, the risk for developing breast cancer by age 70 years is 57% in BRCA1 mutation carriers and approximately 49% in BRCA2 mutation carriers. The risk for developing cancer and associated mortality decreases with age. For example, a 55-year-old woman has an approximately 35% risk for developing breast cancer and a 6% mortality, compared with a 25- year-old woman who has a 65% chance of developing breast cancer with a 14% mortality. This being said, it is important to consider to benefits of prophylactic mastectomies beyond mortality, including patient-specific factors such as the desire to decrease screening and seeking peace of mind.

2023

116
Q

A 37-year-old woman undergoes left breast reconstruction with a deep inferior epigastric artery perforator flap. During dissection of the internal mammary vessels, a small tear is inadvertently made in the left parietal pleura. The decision is made to repair the tear primarily, using a single figure-of-eight suture. Which of the following ventilatory maneuvers is most appropriate immediately prior to cinching the stitch for pleural closure?

A) Decreasing positive end-expiratory pressure
B) Disconnecting the endotracheal tube
C) Pausing at end-expiration
D) Sustaining maximal inspiration
E) Switching to volume-controlled ventilation mode

A

The correct response is Option D.

The most appropriate ventilatory maneuver to perform just before cinching the last stitch of the repair of an isolated parietal pleura tear is sustaining maximal inspiration for a few seconds, also known as a modified Valsalva maneuver. Once the parietal pleura is closed, normal lung ventilation can resume. The goal is to fully inflate the lungs to maximize positive pressure in the pleural cavity, displacing any residual pneumothorax through the pleural defect before finishing an air-tight repair of the parietal pleura or chest wall. The maneuver is probably most commonly performed manually by the anesthesiologist (using the ventilator’s built-in bag valve mask), but it can also be done by holding ventilation at the end of an inspiratory cycle or by significantly increasing the positive-end-expiratory pressure (PEEP).

Pausing ventilation at the end of expiration (without first increasing PEEP), disconnecting the endotracheal tube, decreasing PEEP, or switching to volume-controlled ventilation would not achieve the goal of increasing/maximizing pleural pressure.

2023

117
Q

A 33-year-old woman presents with a drug-resistant gram-positive bacterial infection of an implant after undergoing implant-based breast reconstruction. Which of the following mechanisms is most likely responsible for this bacterial drug resistance?

A) Increased carotenoid content in cell membrane
B) Increased esterification of cell membrane lipoteichoic acid
C) Inhibition of protease expression
D) Reduction of biofilm
E) Suppression of binding ligands against host defense peptides

A

The correct response is Option B.

Bacterial resistance to host defense peptides is an unfortunate reality that is clinically and experimentally observed. The significant diversity of host defense peptide mechanisms requires the resistance strategies to use equally numerous solutions based on the specific peptide or target organism.

Common resistance mechanisms used by bacteria against host defense peptides include:
1) decreased carotenoid content in cell membranes (not increased)
2) increased protease expression (not inhibition)
3) biofilm production (not reduction)
4) increased expression of binding ligands to host defense peptides (not suppression)

Esterification of cell membrane lipoteichoic acid with D-alanine is a commonly used mechanism of gram-positive bacterial antimicrobial resistance.

2023

118
Q

A 28-year-old woman presents with a 4-cm mass of the left breast. Examination of a specimen obtained on biopsy discloses a benign phyllodes tumor. Which of the following is the most appropriate treatment for this lesion?

A) Intensity-modulated radiation therapy
B) Mastectomy with sentinel lymph node biopsy
C) Neoadjuvant chemotherapy
D) Wide local excision
E) Observation

A

The correct response is Option D.

Phyllodes tumors are uncommon fibroepithelial masses. They are classified as benign, borderline, or malignant based on histology. Local recurrence risk is not insignificant. The standard of care includes wide local excision with a clear margin. Although the recommended surgical margin is an area of some controversy, many authors suggest at least a 1-cm rim of normal tissue surrounding the tumor. Very large phyllodes tumors might necessarily lead to mastectomy. However, the patient in this question has a smaller tumor, which would not require mastectomy. Sentinel lymph node biopsy might be considered for a malignant phyllodes tumor, but it is not indicated for a benign tumor. Surgical extirpation of phyllodes tumors may be quite extensive and therefore can present significant challenges to the reconstructive surgeon. There are no data to recommend chemotherapy or radiation therapy in the setting of a benign phyllodes tumor. Observation of the primary tumor is not recommended.

2023

119
Q

A 72-year-old man undergoes wide local excision of a T2 N0 squamous cell carcinoma in the left lateral border of the tongue, along with left cervical lymphadenectomy including sublevels IB and IIA and levels III and IV. The left internal jugular vein, sternocleidomastoid muscle, and spinal accessory nerve are preserved. Which of the following is the most appropriate classification of this neck dissection?

A) Extended
B) Modified radical
C ) Radical
D) Selective

A

The correct response is Option D.

Removal of sublevels IB and IIA and levels III and IV is classified as a selective neck dissection.

Radical neck dissection was first described in 1906 by Dr. George Crile. It includes removal of cervical lymph nodes from levels I through V, along with the ipsilateral internal jugular vein, sternocleidomastoid muscle, and spinal accessory nerve.

Modified radical neck dissection refers to the removal of all cervical lymph nodes routinely included in a radical neck dissection (levels I through V), while preserving at least one of the nonlymphatic structures (internal jugular vein, sternocleidomastoid muscle, or spinal accessory nerve).

Selective neck dissection refers to a cervical lymphadenectomy that preserves at least one of the lymph node levels that are routinely removed in a radical neck dissection (I through V). Levels and sublevels are selected for removal based on the expected drainage pattern determined by the location of the primary tumor. Since many different selective neck dissections are possible, the recommended name of the procedure should include “selective neck dissection” followed by the list of levels or sublevels to be removed, as in: “left selective neck dissection (sublevels IB and IIA and levels III and IV).”

Extended neck dissection refers to the removal of additional lymph node groups (eg, paratracheal) or nonlymphatic structures (eg, hypoglossal nerve) that are not routinely included in a radical neck dissection.

2023

120
Q

A 45-year-old woman with newly diagnosed invasive lobular carcinoma of the right breast comes to the clinic to discuss immediate breast reconstruction following bilateral mastectomy. History includes a sulfonamide allergy; otherwise, she has no medical comorbidities and does not smoke cigarettes. Staged implant-based breast reconstruction is planned. Which of the following prophylactic antibiotic regimens is most appropriate for this patient?

A) Intravenous cefazolin for less than 24 hours perioperatively followed by no oral
antibiotic
B) Intravenous clindamycin for less than 24 hours perioperatively, followed by a 5-day
course of oral trimethoprim-sulfamethoxazole
C) Intravenous clindamycin for less than 24 hours perioperatively, followed by no oral
antibiotic
D) Intravenous piperacillin/tazobactam for less than 24 hours perioperatively, followed
by a 5-day course of oral ciprofloxacin
E) Intravenous vancomycin for less than 24 hours perioperatively, followed by a 5-day
course of cephalexin

A

The correct response is Option A.

Postoperative surgical site infection (SSI) following implant-based breast reconstruction is a major complication that can mandate device removal and thereby result in reconstructive failure. As such, meticulous technique, maintenance of a sterile surgical field, and perioperative antibiotic prophylaxis are of utmost importance in decreasing postoperative infection rates.

Historically, antibiotics were used quite liberally following implant-based breast reconstruction with prophylaxis implemented for an extended duration, frequently until drain removal. More recent studies, however, have demonstrated that extended postoperative systemic antibiotics have no significant effect on decreasing the incidence of SSI. Importantly, Phillips et al demonstrated that antibiotic prophylaxis for an extended duration, versus less than 24 hours, was associated with a greater rate of expander loss. Therefore, contemporary recommendations call for antibiotic prophylaxis limited to the perioperative period only, for less than 24 hours.

As to antibiotic choice, the use of beta-lactam antibiotics (eg, cefazolin) was recently demonstrated as superior to alternative antibiotics with a bacteriostatic mechanism of action (eg, vancomycin, clindamycin) in regards to rates of postoperative infection and reconstructive failure following immediate implant-based breast reconstruction. Trimethoprim-sulfamethoxazole would be contraindicated in this patient given her sulfonamide allergy. Since perioperative antibiotic prophylaxis is predominantly targeted against gram-positive bacteria (skin flora), intravenous piperacillin/tazobactam would not be an appropriate choice.

2023

121
Q

A 62-year-old woman with invasive ductal carcinoma of the left breast is scheduled to undergo a left-sided mastectomy with immediate reconstruction using a free deep inferior epigastric perforator flap. BMI is 30.2 kg/m2 . Preoperative examination shows bilateral lower extremity varicose veins. She has a right chest port that has been used for neoadjuvant chemotherapy. Chemoprophylaxis using a weight-based dose adjustment of which of the following drugs is most appropriate to decrease this patient’s risk for postoperative venous thromboembolism?

A) Intravenous heparin
B) Oral apixaban
C) Oral rivaroxaban
D) Subcutaneous enoxaparin
E) No chemoprophylaxis is indicated

A

The correct response is Option D.

Venous thromboembolism (VTE) is a life-threatening postoperative complication. Numerous studies have demonstrated that chemical prophylaxis is impactful for high-risk plastic surgery inpatients. The patient presented here is a high-risk patient, given that she is an obese woman with a cancer diagnosis, varicose veins, and central venous access, and she is scheduled to undergo a surgical procedure lasting several hours. Hence, chemical prophylaxis is indicated.

Enoxaparin, a low-molecular-weight heparin, has been demonstrated to prevent VTE in high risk plastic surgery patients. However, it has not been uniformly effective with breakthrough VTE events occurring in 4 to 10% of the highest risk patients. This has been attributed to the “one-size-fits-all” approach to enoxaparin prophylaxis (ie, the same dose is administered in all patients), irrespective of body weight and extent of surgery.

Recent level I evidence demonstrated that for plastic surgery inpatients, weight-based enoxaparin administration showed superior pharmacokinetics for avoidance of both under anticoagulation and over-anticoagulation in postoperative patients receiving prophylactic enoxaparin.

Intravenous heparin is not indicated for VTE prophylaxis, nor do studies support its use for routine management in free flap surgery.

Apixaban and rivaroxaban are not indicated for VTE chemoprophylaxis in plastic surgery patients. Indications for their use include:

  • Decrease in the risk for stroke and systemic embolism in nonvalvular atrial fibrillation
  • Prophylaxis of deep venous thrombosis following hip or knee replacement surgery
  • Treatment of deep venous thrombosis/pulmonary embolism
  • Decrease in the risk for recurrence of deep venous thrombosis/pulmonary embolism

2023